429 final practice questions

¡Supera tus tareas y exámenes ahora con Quizwiz!

Complex change situations require that the change leader promote ongoing visioning among staff members. One strategy is to: a. Consciously evaluate invisible mental models. b. Allow for individual outcomes. c. Encourage cooperative activities. d. Operate between order and disorder.

ANS: A Senge's theory on change suggests that each individual or organization bases activities on a set of assumptions, or a set of beliefs, or mental pictures about the way that the world should work. When these invisible models are uncovered and consciously evaluated, it is possible to determine their influences on work accomplishment.

2. Two staff nurses are arguing about whose turn it is to work on the upcoming holiday. In trying to resolve this conflict, the nurse manager understands that interpersonal conflict arises when: a. risk taking seems to be unavoidable. b. people see events differently. c. personal and professional priorities do not match. d. the ways in which people should act do not match the ways in which they do act.

ANS: B By definition, conflict involves a difference in perception between two or more individuals.

10. Two nurses on a psychiatric unit come from different backgrounds and have graduated from different universities. They are given a set of new orders from the unit manager. Each nurse displays different emotions in response to the orders. Nurse A indicates that the new orders include too many changes; Nurse B disagrees and verbally indicates why. This step in the process is which of the following in Thomas' stages of conflict? a. Frustration b. Conceptualization c. Action d. Outcomes

ANS: B Thomas' Stages of Conflict include conceptualization, which involves different ideas and emphasis on what is important or not or about what should occur.

First-order change

Evolutionary change that occurs in planned and small steps. Often referenced as continuous improvement.

The state of being emotionally impelled, demonstrated by a sense of passion and dedication to a project or event, describes: a. commitment. b. control. c. willingness to cooperate. d. communication.

ANS: A Commitment is described as having a sense of passion and commitment to a project without necessarily having a need to control.

22. Knowing when to have the entire team participate in the decision-making process or when to have only the team leader make the decisions depends on the situation and the desired outcomes. The autocratic process is used in which of the following situations? a. The task and the outcome are relatively simple and defined. b. It is unlikely that the group will reach a consensus. c. A decision has to be discussed thoroughly. d. A number of options need to be considered.

ANS: A An autocratic style is appropriate when rapid decision making is required and in situations where the task and the potential outcome are well-defined.

Which ethical principle is primarily involved in informed consent? a. Veracity b. Autonomy c. Beneficence d. Nonmaleficence

ANS: B Autonomy refers to the right to choose freely, which is inherent in informed consent.

You are the nurse manager of a nursing service organization that provides around-the-clock care to clients in their homes. To achieve maximum reimbursement for a client who is recovering from a hip replacement, the nursing staff most likely will follow the nursing care guidelines presented in the: a. nursing care plan. b. physician's orders. c. critical pathway. d. clinical practice guidelines.

ANS: C A critical pathway outlines outcomes, clinical standards, and interventions for a patient in each phase of treatment. The goal of critical pathways is effective coordination of care across various staff and levels of care.

1. Nurses entering the work force today are faced with which of the following relationships that could create organizational conflict? (Select all that apply.) a. Nurse-physician relationship b. Nurse-nurse relationship c. Nurse-patient relationship d. Nurse-chief nursing officer relationship e. Nurse-auxiliary personnel relationships

ANS: A, B, C, D, E By nature, conflict is potentially present in all interpersonal situations. The nurse manager should create an environment that recognizes and values differences in staff, physicians, patients, and communities.

2. How is the action of advocacy described? A) Informing others of their rights and making certain they have sufficient information to make decisions B) Learning about the need for a decision and then making a good decision for other people C) Supporting an individual's right to make a decision even when they do not have accurate information D) Protecting the rights of patients in accordance with the law

Ans: A Feedback: The advocate informs others of their rights and makes certain they have sufficient information to make decisions. The remaining options are incorrect when describing the action associated with advocacy.

6. The chief nursing officer plans a series of staff development workshops for the nurse managers to help them deal with conflicts. The first workshop introduces the four stages of conflict, which are: a. frustration, competition, negotiation, and action. b. frustration, conceptualization, action, and outcomes. c. frustration, cooperation, collaboration, and action outcomes. d. frustration, conceptualization, negotiation, and action outcomes.

ANS: B Thomas (1992) determined that conflict proceeds through these four stages in this particular order.

When goals/outcomes are somewhat unclear in early preparation for a complex change, the manager and the change management team develop several acceptable goals/outcomes. This change in management approach is termed: a. Unfreezing. b. Nonlinear. c. Cybernetic. d. Linear.

ANS: B While Lewin's theory was designed to describe planned or first-order changes, many scholars think the theory is too simplistic to address how unplanned or second-order change occurs. In complex situations with an uncertain change environment, a nonlinear approach that involves flexibility improves overall outcomes. Linear change is appropriate to stable, less complex, and more predictable situations

The mediator suggested to the unit staff that a group agreement needed to be made so meetings could become productive. For example, the group agreement, "We will speak supportively," prevents: a. expression of opposing ideas. b. gossip and making negative comments about absent team members. c. efforts to ensure that everyone thinks alike. d. votes that oppose motions.

ANS: B This is an example of a rule that a team can implement to prevent certain negative behaviors such as gossip, backbiting, and bickering that undermine the productivity and functioning of a group.

A young male nurse began in nursing as a staff nurse at a hospital. After 3 years, he moved to a home healthcare agency for increase in pay and taking a position as a manager in the agency. What type of motivation does this exemplify in career success? a. Internal b. External c. Spiral d. Entrepreneurial

ANS: B This type of career success is exemplified by external focus or motivation. The motivation in external focus is from tangible measures such as salary and promotions.

Literature on oppression in nursing has: a. Verified the presence of behaviors associated with oppression within nursing. b. Suggested that nurses are oppressed because of the actions of other groups. c. Failed to establish that oppression is present in nursing groups. d. Indicated that nurses use oppression negatively.

A

The workgroup on NU 23 is marked by apathy to the ward's patients, high absenteeism, open conflict among team members, and high turnover of personnel, including managers. The underlying behavior in this situation may be characterized as: a. Powerlessness. b. Anger. c. Apathy. d. Oppression.

A

The nurse manager frequently interacts with staff and other hospice facility employees. Communication is purposeful because the manager assesses current issues, such as specific satisfactions and dissatisfactions with the newly implemented computerized documentation system. Informally, the manager gathers available staff members to address similar learning needs. Many times, staff members are found coaching other staff about improving use of the new system. According to Senge (1990), the activities demonstrated in this example are: a. Dialogue, team learning. b. Resilience, personal mastery. c. Shared vision, systems thinking. d. Mental models, teachable moments.

ANS: A Building shared vision occurs when leaders involve all members in moving personal visions of the future into a consolidated vision common to members and leaders.

5. Sarah, a staff nurse on your unit, witnesses another nurse striking a patient. Sarah wants to remain friends with her colleague and worries that confrontation with her colleague or reporting her colleague will destroy their relationship. Sarah is experiencing which type of conflict? a. Intrapersonal b. Interpersonal c. Organizational d. Professional

ANS: A Intrapersonal conflict occurs within a person when confronted with the need to think or act in a way that seems at odds with that person's sense of self. Questions often arise that create a conflict over priorities, ethical standards, and values. Some issues present a conflict over comfortably maintaining the status quo and taking risks to confront people when needed, which can lead to interpersonal conflict.

An individual in a wheelchair is applying for the position of receptionist in an outpatient clinic. The nurse manager understands that the Americans with Disabilities Act of 1990 requires that employers: a. Make reasonable accommodations for persons who are disabled. b. Allow modified job expectations for persons recovering from alcoholism. c. Hire disabled individuals before hiring other qualified, non-disabled persons. d. Treat, for purposes of employment, homosexuals and bisexuals as disabled.

ANS: A The purposes of the ADA are to eliminate discrimination against persons with disabilities and to provide consistent, enforceable standards to address discrimination in the workplace. The Americans with Disabilities Act of 1990 provides protection to persons with disabilities and is the most significant civil rights legislation since the Civil Rights Act of 1964. the purpose is to provide a clear and comprehensive national mandate for the elimination of discrimination against individuals with disabilities and to provide clear, strong, consistent, enforceable standards addressing discrimination in the workplace.

4. The nurse on the 7-7 shift is assigning a specific component of care to an unlicensed nursing personnel (UNP) employee. The night nurse would remain: a. accountable. b. responsible. c. authoritative and liable. d. responsible and task-oriented.

ANS: A When a registered nurse delegates care to a UNP, responsibility is transferred; however, accountability for patient care is not transferred. Thus, "accountability rests within the decision to delegate while responsibility rests within the performance of the task" (Anthony and Vidal, 2010, p. 3).

From the information presented in this chapter, which of the following statements best defines an accomplished team? Effective teams: a. can create a form of synergism in which the outcome is greater than the sum of the individual performances. b. do not necessarily need goals, objectives, vision, and a clearly stated purpose. c. do not always have effective communication patterns. d. may or may not have a clear plan that is followed and revisited and has an ongoing evaluation scheme.

ANS: A Research on team building over many years has established that high-functioning teams are characterized by synergy that takes the team from a collection of individuals to an outcome that is greater than the sum of the parts.

A conflict develops between an associate nurse and a primary nurse over the assessment of a patient with pulmonary edema. Based on her assessment of the patient, the associate nurse insists that it is her role to change the care plan because she is the one who has made the assessment. As the nurse manager, you clarify that: a. it is the role of the primary nurse to make alterations based on assessment data and input. b. the associate nurse is accountable and responsible while the primary nurse is off duty and therefore is able to alter the care plan. c. neither the primary nor the associate should make changes without first consulting you as the manager. d. it really does not matter who alters the nursing care plan as it depends on situation and time to do so.

ANS: A The primary nursing model assumes that the primary nurse is accountable for patient care, even while off duty. The primary nurse is responsible for establishing the patient plan of care and therefore for altering it with the input of an associate. The associate is responsible for implementation of the established care plan when the primary nurse is off duty.

Based on Elizabeth's insights and suggestions, you involve pharmacy, only to discover that the change in practice involves practice committees, a medical practice committee, and concerns from administration about potential costs and safety of the proposed change to the IV protocols. The change process at this point is: a. Linear. b. Nonlinear. c. Sabotaged. d. Neutralized.

ANS: B Complex change involves nonlinear processes and a variety of strategies to negotiate influences on change. Complexity theories alter the traditional systems thinking approach by asserting that system behavior is unpredictable. This theory views change as emergent, nonlinear, and highly influenced by all individuals and subsystems in an organization

10. An outpatient surgery manager is evaluating new infusion pumps for purchase to use in the operating room. The manager should: a. select the least expensive brand. b. use a decision-making tool to evaluate brands. c. ask the nursing staff which brand they prefer. d. select the vendor the institution usually buys from.

ANS: B Decision-making tools such as decision grids and SWOT analyses are most appropriate when information is available and options are known.

22. Functions such as "delegates tasks to assistive personnel" that are outlined in a position description for an RN Team Leader would be considered: a. active delegation. b. passive delegation. c. passive accountability. d. active responsibility.

ANS: B Delegation of functions that are normally considered part of or an essential part of the practice of a licensed person through a position description is considered passive delegation.

A nurse belongs to several professional organizations, serving on a state-level committee of one group and on two task forces at work. The nurse is committed to a range of health issues. This nurse exemplifies which level of political activism in nursing? a. Gladiator b. Buy-in c. Self-interest d. Political sophistication

D

During a unit meeting, you notice that Vivian listens attentively when Mary is speaking and offers support and advice when Mary presents ideas to the group. You are surprised because Vivian has often confided that she does not like Mary. Vivian's behavior is best described as: a. Insincere. b. Networking. c. Politically sophisticated. d. Collegial.

D

Lucy, head nurse on the surgical unit, works with her staff to find ways in which they can work together with other disciplines to provide more effective care for patients on the unit. Lucy likely knows her power is: a. Limited, thereby necessitating involvement of others in implementing ideas. b. Restricted, which necessitates finding alternative means to achieve strong patient outcomes. c. Directly primarily toward those who are subordinate to her. d. Of unlimited capacity when shared with others.

D

Five organizational disciplines to support change and evolution

Shared vision, mental models, personal mastery, team learning, systems thinking.

During review of back injuries, it is determined that a large number of injuries are occurring in spite of mechanical lifts being used. Furthermore, it is determined that some lifts are outdated. In addressing this concern, the unit manager: a. Meets individually with nurses who are observed to be using the lifts incorrectly to review the correct procedure. b. After consultation with the staff about the review, orders new lifts to replace older ones that are malfunctioning. c. Blames the system for inadequate funding for resources. d. Reviews the system of reporting incidents to ensure that appropriate reporting is occurring.

a. Meets individually with nurses who are observed to be using the lifts incorrectly to review the correct procedure. The IOM report (2004) points to the need to involve nurses in decisions that affect them and the provision of care.

In preparation for redesignation as a Magnet Hospital, how would you prepare? a. Commit staff resources over a 6-month period to updating procedure manuals. b. Educate staff through meetings and training sessions regarding appropriate answers to questions. c. Prepare a manual that outlines orientation procedures and ensure that all safety issues are addressed. d. Ensure that there are empirical data to support review of patient outcomes, actions taken, and results of actions.

d. Ensure that there are empirical data to support review of patient outcomes, actions taken, and results of actions. Through the MagnetTM model, organizations must demonstrate how they provide excellence in five areas. Between designation and redesignation as a MagnetTM organization, greater emphasis is placed on empirical quality results.

The chief nursing officer has asked the staff development coordinator to facilitate the development of a clinical competency program for the facility. While making rounds on the units, the staff development coordinator overhears RN staff complaining that they feel it is insulting to be required to participate in a competency program. Which behavior by the staff development coordinator is most appropriate in this situation? a. Disregard staff concerns and continue with development of the program. b. Inform the nurses that this program is a requirement for JCAHO accreditation. c. Schedule a meeting with the chief nurse executive to apprise her of the situation. d. Facilitate a meeting so nurses can articulate their values and concerns about a competency program.

d. Facilitate a meeting so nurses can articulate their values and concerns about a competency program. The manager role involves guiding others through a set of derived practices that are evidence-based and known to satisfy preestablished outcomes such as participation in a competency program. This involves engagement of staff through sharing of concerns and ideas. A close analysis of the IOM report and the summary of the PPACA suggests that no health reform can unfold without active nursing engagement. Each document emphasizes that nurses must lead, manage, and behave as active collaborators with other members of the health team and with those being served.

"Power in nursing" refers to the nurse's ability to: a. protest unfair working conditions through walkouts and strikes. b. demonstrate knowledge about organizational behavior. c. act on issues that influence nursing licensure but not patient care. d. use one's influence to create change in pursuit of goals.

use one's influence to create change in pursuit of goals.

Sarah wonders about the direction that you have given regarding management of incontinent, confused patients. She brings you evidence that she has found regarding incontinence interventions and asks you if she and you could talk about the guidance that you have given after you have had an opportunity to read the articles she has given you. This is an example of (select all that apply): a. Assertiveness. b. Followership. c. Management. d. Insubordination.

a. Assertiveness. b. Followership. This is an example of followership in which a staff nurse is demonstrating assertive behavior and presenting evidence that may influence the decision making of her nurse leader and manager.

One day, at coffee, your co-worker suggests that you and she sit with unit members of the hospital research committee. She suggests that this would be an excellent way to get to know people who share her interest in research. Her actions are an example of: a. Mentorship. b. Politics. c. Networking. d. Empowerment.

C

As a new manager in the ED, you meet with each of the staff to ask about their priorities and what they think is going well in the department or what is of concern to them. Almost all of the staff express frustration and distress at being treated rudely or disrespectfully by patients, staff from other departments, and physicians and complain that they feel that nurses in the ED are not valued. With the staff, you brainstorm to raise the profile of nurses. Which of the following strategies would be most effective? (Select all that apply.)

*Speaking positively about one's work *Dressing and grooming in a clean and neat manner *Using titles (e.g., Mr., Mrs., Ms.) and last names *Submitting a written complaint to senior *Developing a code of conduct for the ED staff.

A staff nurse asks the nurse manager for a few days off for personal reasons. The nurse manager turns in the request to the human resources office with a note indicating that the staff nurse has demonstrated excellent working skills and is a valued employee. The nurse manager has used the influence of her position to help this staff member. Influence is the process of: a. Using power. b. Empowering others. c. Understanding power. d. Moving past apathy.

A

During orientation of new nurse managers, the chief nursing officer stresses strategies that help nurse managers to achieve a powerful image. Which groups of behaviors best contribute to a powerful image for the nurse manager? a. Greeting patients, families, and colleagues with a handshake and a smile; listening carefully when problems arise b. For men, no facial hair, always wearing a suit and tie; for women, always wearing a suit and high-heeled shoes c. Maintaining a soft voice during times of conflict; making unbroken eye contact during interactions d. Smiling all the time; always wearing a suit and carrying a briefcase; women should wear no jewelry

A

A nurse on your inpatient psychiatric unit is found to have made sexually explicit remarks toward a patient with a previous history of sexual abuse. The patient sues, claiming malpractice. Which of the following conditions may not apply in this situation? a. Injury b. Causation c. Breach of duty d. Breach of duty of care owed

ANS: A (Elements of malpractice: Duty owned the patient: failure to monitor a patient's response to treatment. Breach of the duty owed: failure to communicate change in patient status to the primary healthcare provider. Foreseeability: failure to ensure minimum standards are met, Causation: failure to provide adequate patient education. Injury: fractured hip and head concussion after a patient fall. Damage: Additional hospitalization time, future medical and nursing care needs and costs. ) By virtue of employment, the nurse owes a duty of care to the patient; this care has been breached by a nurse, who would be expected to know that this behavior violates usual standards of care. The resultant injury, the fifth malpractice element, must be physical, not merely psychological or transient. In other words, some physical harm must be incurred by the patient before malpractice will be found against the healthcare provider, which is not evident in this situation where the action did not involve physical harm.

As a new manager, you are shocked to learn that your unit is still using heparin in heparin locks. You are aware of evidence related to this practice and want to change this practice as quickly as possible on your unit. You are in which stage of Lewin's stages of change? a. Unfreezing b. Experiencing the change c. Moving d. Refreezing

ANS: A Although you may be at a higher level of change in relation to your individual practice and knowledge of the use of change, in this situation, you are recognizing the need for change in relation to practice on the unit that you are managing. This phase is the initial phase in first-order change and will involve listening to staff to see if they perceive a similar problem.

9. The risk manager wants to evaluate the reasons for an increased number of falls on the rehab unit. The risk manager devises a fishbone diagram. A fishbone diagram is a useful tool to: a. identify the root causes of problems. b. list possible solutions to problems. c. help leaders select the best options. d. evaluate the outcomes of decisions made.

ANS: A A fishbone diagram, also known as a cause-and-effect diagram, is useful for determining the reasons (causes) for an effect (falls).

11. Select the statement that best defines the difference between problem solving and decision making: a. decision-making skills require critical thinking; problem-solving skills do not. b. problem-solving skills require critical thinking; decision-making skills do not. c. decision making is a goal-directed effort; problem solving is focused on solving an immediate problem. d. problem solving is a goal-directed effort; decision making is focused on solving an immediate problem.

ANS: C Problem solving is focused on solving immediate problems, whereas decision making is a goal-directed process that is aimed at selecting appropriate actions from among options. Not all decisions begin with a problem.

21. County Hospital has position descriptions for all staff, including RN Team Leaders. Sarah, a team leader on the rehab unit, assesses the needs of the patients in her area, assesses the skills and backgrounds of each of the individuals on her team, and then assigns and delegates the appropriate care provider to each patient and task. Sarah's activity in the example described is termed: a. passive delegation. b. passive accountability. c. active delegation. d. active responsibility.

ANS: C When a position description contains functions that are considered to be the normal practice of the person in that role, then it is considered a passive delegation act. When Sarah decides what is best for the patients in her care in terms of who should perform the care and then holds the person accountable, she is engaging in active delegation.

As a nurse manager, you notice that one of your new nurses has provided exceptional care for a patient with especially complex needs. What would be the MOST effective way of recognizing the nurse's performance? a. At the next performance review, note specifically what the nurse did to make the patient comfortable. b. To avoid embarrassing the nurse in front of others, find a way to compliment the nurse in private. c. When the nurse comes out of the room, tell the nurse specifically what you appreciated about the care that was provided. d. Encourage the patient to note the care on the patient feedback form so that the institution can recognize the nurse's efforts.

ANS: C Acknowledgement is most effective when it is specific, timely, given in public, sincere, and on an eye-to-eye basis. The more time that elapses between the event and acknowledgement, the less effective it becomes.

The chief nursing officer decided that the nurse managers need a series of staff-development programs on team building through communication and partnerships. She understood that the nurse managers needed to build confidence in ways of handling various situations. The greatest deterrent to confidence is: a. lack of clarity in the mission. b. lack of control of the environment. c. fear that one can't handle the consequences. d. fear that the boss will not like one's work.

ANS: C Fear of not being able to manage consequences undermines confidence and a sense of competency.

An experienced staff nurse applies for a distinct position of patient advocate at a new clinic. Based on the various tools available to her, which of the following should she bring to her interview to best present her skills? a. Résumé b. Annual evaluations c. Portfolio d. Patient advocacy project

ANS: C A portfolio enables prospective employers to view evidence of significant achievements and responsibilities that would be pertinent to the position.

Professional associations: a. set regulatory requirements and establish entry requirements for nursing. b. offer graduate programs for clinical and career advancement. c. provide opportunities for career networking and support. d. are open to all individuals who meet the criteria for membership.

ANS: C Professional associations are frequently, although not always, voluntary groups whose members provide leadership in issues and policies of interest to nurses. Professional associations also offer networks of nurses with similar interests.

13. Jill is the head nurse on a unit in a large hospital. Two of the staff nurses are constantly arguing and blaming each other, and a resolution has not occurred in months. To solve the existing conflict, which is the most creative conflict resolution? a. Avoiding b. Competing c. Compromising d. Collaborating

ANS: D Collaboration, although time consuming, is the most creative stance. The collaboration technique involves both sides in the conflict working together to develop an optimal outcome. This results in a win-win solution.

To satisfy duty of care to a patient, a nurse manager is legally responsible for all of the following except: a. Notifying staff of changes to policies related to medication administration. b. Scheduling and staffing to ensure safe care. c. Delegating in accordance with practice acts. d. Supervising the practice of the physician.

ANS: D Legally, the nurse manager is accountable to nursing practice standards, standards for nurse administrators, and hospital policies and procedures.

15. Sarah is a nurse manager in a surgical unit. She is concerned about a conflict between Lucy (a staff nurse) and one of the maintenance personnel. Sarah explains to Lucy that unsatisfactory resolution of the conflict is typically destructive and will result in: a. decreased frustration between the maintenance worker and her. b. a good relationship with the maintenance department. c. eventual resolution of the problem without further intervention. d. decreased productivity on her part.

ANS: D Productivity decreases with destructive conflict, whereas constructive conflict strengthens relationships.

13. How can nurses act as advocates? Select all that apply. A) Helping others make informed decisions B) Directly intervening on behalf of others C) Providing consent to treat for the cognitively impaired patients D) Making health-care decisions for those who are not able to do so

Ans: A, B Feedback: Nurses may act as advocates by either helping others make informed decisions, by acting as intermediary in the environment, or by directly intervening on behalf of others. Advocates do not make decisions or provide consent but rather educate patients so they can be informed.

5. Who plays a primary role in ensuring that workers have reasonable, working schedules? A) Unions B) Top administrators C) Managers D) Federal government

Ans: C Feedback: In workplace advocacy, the manager works to see that the work environment is both safe and conducive to professional and personal growth for subordinates. The other options play a role in such situations but the manager has the primary responsibility.

20. Which standard of the American Nurses Association (ANA) Scope and Standards for Nurse Administrators suggests that nurse administrators should advocate for subordinates as well as patients? A) II B) III C) IV D) V

Ans: D Feedback: Standard V of the ANA Scope and Standards for Nurse Administrators suggests that nurse administrators should advocate for subordinates as well as patients. This issue is not addressed in the other options.

A nurse manager is experiencing poor staff morale on her unit. While participating in a baccalaureate course, the nurse manager had learned that one of the reasons nurses lack power today is probably because of the past. In the early decades of the profession, nurses lacked power because: a. Nurses freely chose to defer to physicians and administrators with more education. b. Women lacked legal, social, and political power because of legal and cultural barriers. c. The first nursing licensure laws prohibited nurses from making most decisions. d. Nurses astutely recognized the risks of grabbing too much power too soon.

B

The institution where you are a nurse manager has resisted the adoption of a new document management software, citing cost as a concern. You meet with other nurse managers who are in favor of the software and prepare a proposal to take to the senior executive with the goal of persuading the executive to adopt the software. This is an example of: a. Collaboration. b. A coalition. c. Networking. d. Policy building.

B

Two nurses approach their manager about a conflict regarding the next month's schedule. The nurses are talking loudly and at the same time. The manager most effectively uses communication skills to resolve the conflict by: a. Taking both nurses aside, separately and then together, and charging them with resolving the problem without her direct intervention. b. Listening to each nurse speak to the other without interruption and asking clarifying questions to help them resolve the issue themselves. c. Separating the nurses, instructing each to decide how the problem can be resolved, and meeting with them the next day. d. Calling an emergency scheduling committee meeting and asking volunteers to resolve the conflict between the two nurses.

B

You have been offered a position as a head nurse in the ED. You understand that: a. You have stature and power by accepting the position. b. You have stature but no power at this point in your position. c. As a result of your position, you will need to keep much knowledge to yourself. d. Knowledge is assumed as a result of your position.

B

Your colleague, Mary, a recent graduate announces one day that she intends to leave nursing in 3 to 4 months to pursue a position in marketing. While at your agency, she plans to give patients excellent care and to learn as much as she can, because "Who knows? Nursing is a great job with a great pay and I may return someday." Mary's statements most accurately exemplify which orientation to the concept of nursing? Nursing as a(n): a. Profession. b. Occupation. c. Flexible discipline. d. Career with off and on ramps.

B

Your institution has identified a recent rise in postsurgical infection rates. As part of your QI analysis, you are interested in determining how your infection rates compare with those of institutions of similar size and patient demographics. This is known as: a.Quality assurance. b.Sentinel data. c. Benchmarking. d.Statistical analysis

Benchmarking

Susan, an RN in the ED, would like to pursue leadership roles in her career. She is frustrated that others in her working environment seem to pay little attention to her creative ideas or place her in informal leadership positions. As her colleague, you want to provide her with helpful feedback. Which of the following statements will provide feedback as to how she might communicate power and demonstrate that she is capable of handling other leadership responsibilities? a. "I find your soft voice and manners very reassuring and calming to patients." b. "Try using a wider vocabulary and big words so that people will think that you are knowledgeable." c. "At times, you tend to slump and avoid eye contact when you are talking with colleagues and families." d. "Don't worry about what others think of you. If you feel like saying something, say it, even if it hurts other people's feelings."

C

A staff nurse who was fired for reporting patient abuse to the appropriate state agency files a whistleblower lawsuit against the former employer. Reasons that the court would use in upholding a valid whistleblower suit claiming retaliation include that the nurse: a. Had previously reported the complaint, in writing, to hospital administration. b. Had threatened to give full details of the patient abuse to local media sources. c. Was discharged after three unsuccessful attempts at progressive discipline had failed. d. Had organized, before filing the complaint, a work stoppage action by fellow employees.

ANS: A An employer is unable to fire an employee who, in good faith, reports what is believed to be a violation of a law, rule, or state or federal law. the first exception is a public policy exception, this exception involves cases in which an employee is discharged in direct conflict with established public policy. Under this exception, an employer may not discharge an employee if it would violate the state's public policy doctrine or a state or federal statute, some examples include discharging an employee for serving on a jury, reporting employer's illegal actions (better known as whistle blowing, or the disclosure of information regarding misconduct within a workplace that either is illegal or endangers the welfare of others) and filling a workers' compensation claim. most states and the District of Columbia recognize public policy as an exception to the at will rule. What hack is this question???????

16. Staff at Valley Hospital are concerned that recent staffing cuts will affect their ability to provide quality patient care, and they express their concerns to senior management. The CEO of Valley Hospital makes the following statement: "We need to contain costs because our funding has been decreased." This is a good example of which of the following conditions that propel a situation toward conflict? a. Incompatible goals b. Role conflicts c. Structural conflict d. Competition for resources

ANS: A Conflicts arise in four areas: goals, facts, approaches, and values. Conflicts among goals arise from competing priorities such as the provision of quality patient care and containment of costs.

13. The maintenance department wishes to have the nursing lounge renovated, so the lounge will be more "user-friendly." The department asks the nursing staff to make a wish list of everything that they would like to see in the new lounge. This process is an example of which part of the decision-making process? a. Assessment/Data collection b. Planning c. Data interpretation d. Generating hypotheses

ANS: A In this particular model (a model similar to the nursing model), data collection is the first step toward identifying important alternatives or determining if there is a problem or problems.

A colleague asks you to give her your password access so that she can view her partner's healthcare record. This request violates the patient's right to: a. Privacy. b. Confidentiality. c. Undue authorization of treatment. d. Protection against slander.

ANS: A Privacy refers to the right to protection against unreasonable and unwarranted interference with the patient's solitude. Privacy standards limit how personal health information may be used or shared and mandate safeguards for the protection of health information. Institutions can reduce potential liability in this area by allowing access to patient data, either written or oral, only to those with a "need to know." Persons with a need to know include physicians and nurses caring for the patient, technicians, unit clerks, therapists, social service workers, and patient advocates. Others wishing to access patient data must first ask the patient for permission to review a record. confidentialyity: is the right to privacy of the health record.

As a charge nurse, you counsel your RN staff member that he has satisfied his duty of care by notifying a child's physician of his concerns about deterioration in the child's status at 0330 hours. The physician does not come in. The child dies at 0630 hours. As the charge nurse, you could be held liable for: a. Professional negligence. b. Assault. c. Avoidance. d. Murder.

ANS: A Professional negligence can be asserted when there is failure to do what a reasonable and prudent nurse would do in the same situation. In this situation, the charge nurse might have advocated further for the patient in light of the evident seriousness of the child's condition.

14. Which of the following would be most in line with Hersey model and concepts? a. The team of caregivers on day shift are familiar with their roles and with the patients. The nurse manager decides to work on the unit budget in her office. b. After a year of working on the unit, Shari, an LPN, is still hesitant about many policies and procedures. The charge nurse decides to challenge Shari with more difficult patients. c. The nursing supervisor asks one of her charge nurses to lead a technology integration project. The supervisor continuously demands involvement in decisions that the charge nurse is making in the project. d. Team members complain that Alysha, an RN, is unmotivated, and that she refuses assignments that are complex or difficult. The charge nurse suggests that Alysha is relatively new and that she needs time to adjust.

ANS: A The Hersey model/framework suggests that when ability (skills, job knowledge) and willingness are strong, the involvement of the delegator is less.

14. A good nursing decision maker is one who: a. uses various models to guide the process based on the circumstances of the situation. b. adopts one model and uses it to guide all decision making. c. decides not to use any models because they are all useless. d. develops a new model each time a decision has to be made.

ANS: A The decision model that a nurse uses depends on specific circumstances. Is the situation routine and predictable or complex and uncertain? Is the goal to make a decision that is "just good enough" (conservative) or one that is optimal?

7. After using a mediator to resolve a conflict between the nurse manager and two staff nurses, the chief nursing officer decides to: a. observe to make sure the conflict has been resolved. b. fire both staff nurses. c. reassign both staff nurses. d. reassign the nurse manager.

ANS: A The nurse leader should follow up to determine if the conflict has been resolved because, in professional practice environments, unresolved conflict among nurses is a significant issue that results in job dissatisfaction, absenteeism, and turnover, as well as in decreased patient satisfaction and poorer quality in patient care.

14. A nurse educator is giving a workshop on conflict. During the sessions, he makes various statements regarding conflict. All of the statements are true except: a. conflict can decrease creativity, thus acting as a deterrent for the development of new ideas. b. horizontal violence involves those with similar status but little power in the larger context. c. interprofessional collaboration reduces unresolved conflicts. d. all conflicts involve some level of disagreement.

ANS: A The opposite is true because research has shown that conflict, like change, increases creativity and allows for the development of new ideas.

For a nurse manager in the functional nursing model, an approach that will assist in maintaining staff satisfaction in this specific model is: a. rotation of task assignments. b. frequent opportunities for in-service education. c. orientation to job responsibilities and performance expectations. d. team social events in off hours.

ANS: A Although repetition of tasks increases confidence and competence, it can also lead to boredom. Rotation of tasks can assist specifically in this model to reduce the boredom that is a potential disadvantage of this model.

A patient is admitted to a medical unit with pulmonary edema. His primary nurse admits him and then provides a written plan of care. What type of educational preparation best fits the role of primary nurse? a. Baccalaureate b. Associate c. Diploma d. LPN/LVN

ANS: A Because of the breadth of nursing knowledge required, baccalaureate education is preferred for primary nurses.

The difference between a nurse practitioner's charge of $45 for an office visit and the insurance company's payment of $34 is: a. a contractual allowance. b. a profit. c. a flat rate. d. revenue.

ANS: A Because the amount that is allowed for an office visit is less than the amount that the NP charges, this is known as a contractual allowance or discount. If the amount allowed were more than what the NP charges, then a profit would be realized. All of the answers represent sources of revenue.

The relief charge nurse has assigned a newly licensed baccalaureate-prepared nurse to be one of the team leaders for the 3-11 shift. In making this decision, the charge nurse has overlooked this nurse's: a. clinical expertise. b. leadership ability. c. communication style. d. conflict-resolution skills.

ANS: A Because the basic education of baccalaureate-prepared RNs emphasizes critical-thinking, clinical reasoning, and leadership concepts, they are likely candidates for such roles. Benner (2001), however, identified five stages of clinical competence for nurses: novice, advanced beginner, competent, proficient, and expert. She suggests that competence is typified by a nurse who has been on the job in the same or similar situation 2 to 3 years. Nurses who are at the novice or advanced beginner stage would be less likely than their more experienced counterparts to implement any type of delivery model effectively and thus, this assignment overlooks the nurse's level of clinical expertise and leadership experience.

Which of the following would not be a characteristic of an effective team nurse leader? a. An autocratic perspective b. Excellent communication skills c. Awareness of everyone's abilities d. A genuine interest in team members

ANS: A In ineffective teams, leadership tends to be autocratic and rigid, and the team's communication style may be overly stiff and formal. Members tend to be uncomfortable with conflict or disagreement, avoiding and suppressing it rather than using it as a catalyst for change. When criticism is offered, it may be destructive, personal, and hurtful rather than constructive and problem-centered. Team members may begin to hide their feelings of resentment or disagreement, sensing that they are "dangerous." This creates the potential for later eruptions and discord.

Physicians in a small urban hospital are reluctant to discharge older adult patients because many of the patients lack private insurance and the resources to travel distances for follow-up care. The hospital administration pressures the physicians to discharge patients sooner and to be more consistent with the number of hospitalization days specified within the DRGs. Which of the following would most likely prompt the action of administrators? a. The hospital is incurring a deficit related to a gap between the PPS and the DRGs and costs of care. b. Local home care services are expressing concern about the increased acuity of patients being discharged into their care. c. The resource-based relative scale for physicians does not account for the increased length of stay. d. Acute care patients are being denied entry to the hospital because of the increased stay of patients.

ANS: A Length of stay (LOS) is the most important predictor of healthcare costs and extra days are a cost to the organization in terms of both the extra days and decreased patient volume. The situation, as outlined, does not indicate that there is a bed shortage and therefore, there is no evidence that other patients are being denied access to services or that additional patient volume is not being captured. The hospital would be concerned about the impact on its income because of the additional, uncompensated care costs incurred for patients who exceed the usual length of stay explicitly calculated under PPS and the DRGs.

Of the following, which is the most effective strategy that a nurse manager could employ to reduce unnecessary costs in specific healthcare settings? a. Training nurses on accurate documentation of supplies used for patient care b. Reducing the number of overtime hours worked by staff c. Reducing the number of staff on a unit d. Making decisions for patients about which care is important to their health

ANS: A Reducing overtime hours needs to be carefully assessed against the reasons for overtime (e.g., staff overload, recent illnesses, increased acuity) in terms of whether this reflects an ongoing or temporary situation and therefore whether it is a necessary variance or not. Reducing staff may or may not be effective, depending on the services being provided. Making decisions for the patient misses an important opportunity to invite the patient into discussion about patient priorities and needs and relative costs of options, which may lead to improved adherence with regimens and less waste of resources. An effective approach to cost containment is ensuring that supplies are accurately accounted for and charged in a timely manner.

During times of nursing shortages and increased nursing costs in health care, which of the following nursing care delivery models might come under greatest scrutiny? a. Case method b. Team nursing c. Functional nursing d. Nurse case management

ANS: A The case method may involve total patient care provided by a registered nurse, which, in today's costly healthcare economy, is very expensive. In times of nursing shortages, there may not be enough resources or nurses to use this model.

Because of the complexity of reimbursement systems and its implications for the services available to patients, the nurse has a key role in: a. advocacy for patients with regard to services required and services utilized. b. increasing the volume of services and decreasing the number of patients served. c. accomplishing more with each visit and decreasing the volume of services used. d. decreasing the volume of services used and the number of return visits.

ANS: A The specific strategies employed by organizations and nurses to contain costs and increase revenues depend on the reimbursement system(s) within which the organization operates and therefore on whether the volume of services is increased or whether the volume of services is decreased by placing greater emphasis on efficacy in each visit and reducing the number of return visits. Because of the complexity of the reimbursement environment, the nurse is placed in a position of advocacy in terms of what the patient needs and how those needs can be best met within the funding structures.

An example of an initiative that may reduce total healthcare costs would be: a. offering nurse practitioner-led clinics that educate parents about non-pharmacologic strategies for managing ear infections. b. educating seniors about the comparative costs of medications that are prescribed to them. c. lowering copayments for prescription drugs for seniors. d. advocating for more readily available MRI services to ensure early diagnosis.

ANS: A Total healthcare costs are a function of prices that are established for various services and the volume or quantity of services used. Utilization of high-tech diagnostic services and lowering of copayments have been implicated in increasing total healthcare costs (thus C and D would not be correct), as well as attitudes and behaviors of consumers of health care. In general, consumers prefer to "be fixed" when something goes wrong rather than to practice prevention. Many consumers still believe that the physician knows best, so they do not seek much information related to costs and effectiveness of different healthcare options. When information is sought, it is not readily available or understandable. Also, consumers are not accustomed to using other, less costly healthcare providers, such as nurse practitioners.

To reduce reliance on overtime hours, an organization develops a strategy for floating nurses during staff shortages. To maximize patient safety and reduce costs, the healthcare organization: a. develops a centralized pool of float nurses. b. assigns nurses from less busy units to ones with increased acuity levels. c. floats nurses only between units on which the nurses have been cross-trained. d. assigns float nurses to basic care only.

ANS: A A centralized pool usually includes experienced nurses who maintain a broad range of competencies. Other approaches are less satisfying for nurses, are less efficient, and may be less safe.

The primary difference between a résumé and a CV is that a résumé: a. reflects your skills, knowledge, and background in relation to a specific position. b. offers a detailed listing of positions held and where positions were held. c. includes a long and detailed explanation of academic and work experience. d. provides contact information and focuses on your background, in general.

ANS: A A résumé is a short, customized overview of your professional life that relates to the qualifications of specific positions and how you are able to match your background to the qualifications that are desired. Provision of contact information is common to both the résumé and the CV. Résumés are more effective if details of particular positions that have been held are highlighted as compared with a detailed listing of positions held.

Knowing your professional strengths is important to: a. find your fit in positions and a career path. b. maintain a professional status. c. act in a manner that is legal and ethical. d. understand the role expectations of a position.

ANS: A Being aware of your strengths is critical in determining what you will bring to a position and can be used to find your fit and possible career path. Knowledge and experience are important in maintaining the privilege of belonging to a profession and of behaving ethically and legally

A functional résumé focuses on: a. experience and skills gained in positions. b. positions held and specific roles in the positions. c. academic qualifications and achievements. d. relating skills and experience to qualifications in a specific position.

ANS: A Functional résumés highlight skills and experience gained rather the details of specific positions. As with résumés in general, skills and experiences are customized to create an image of an individual in a particular position.

As the unit manager, you post the staffing plan and compliance reports. This initiative is aimed at: a. maintaining unit morale. b. complying with national requirements. c. demonstrating patient outcomes. d. inviting staff participation in decision making

ANS: A Hospitals are responsible for monitoring the extent to which actual staffing matches the staffing plans, making revisions as necessary. The Joint Commission accreditation reviews staffing plans against any obvious staffing deficiencies and patient care concerns. Posting of the staffing plan is required in some states so that staff may view it. Adequate staffing, as demonstrated through a staffing plan, and compliance reports contribute to staff morale.

To maintain patient safety, studies suggest that scheduling should avoid: a. rotating shifts. b. weekends. c. 8-hour shifts. d. mandatory overtime.

ANS: A Rotating shifts and overtime past 12 hours (mandatory or not) are being shown to increase nurse error and jeopardize patient safety.

Steady state styles would be most likely in which of the following situations? a. Small hospital, in an isolated rural setting, with limited hierarchy b. Large urban teaching hospital c. Health network with several organizations d. Travel nurse agency

ANS: A Steady state career styles (career-long commitment to a particular position) are more likely in rural settings, where commitment to the community is high and alternative career opportunities are limited.

With regard to nursing practice, nurse managers are held responsible for (select all that apply): a. Practicing within legal guidelines established under state law and nurse practice acts. b. Ensuring that nursing staff under their supervision are currently licensed to practice. c. Referring all errors in nursing judgment to state discipline boards. d. Ensuring that physicians are properly licensed to provide care on patient care units.

ANS: A, B Nurses are responsible for knowing and practicing under state law and nurse practice acts. Managers are responsible for monitoring staff practice and ensuring that staff hold current, valid licensure.

One of your staff nurses asks for your advice because a patient refuses to sign a consent for surgery. The patient says that he won't sign because he doesn't understand the nature of the surgery. You advise that (select all that apply): a. Consent must not be coerced. b. The patient has a right to choose not to consent. c. The patient must sign the consent because the doctor wants him to sign. d. Witnessing a consent is related only to the voluntary nature of the signature.

ANS: A, B, D Consent must be voluntary and not coerced; the patient must understand what he is signing, must have legal capacity, and must understand the consequences of refusal. Witnessing a consent means attesting to the voluntary nature of the patient's signature.

1. An example of a nursing care activity that would not be delegated by an RN to a UNP is: (Select all that apply.) a. teaching self-catheterization to a patient with paraplegia who has limited English. b. basic care for a patient with a head injury who is rapidly deteriorating. c. one-to-one observation with a suicidal patient. d. assessment of patients being admitted through the Emergency Department. e. basic hygienic care for a patient who is post MI and stable.

ANS: A, B, D Functions such as assessment, diagnosis, planning, and evaluation cannot be delegated. In addition, stability, critical thinking, time, and safety are factors that are considered in assessing whether or not to delegate care to a UNP. Teaching self-catheterization to a patient with limited English requires critical thinking; basic care for a patient who is rapidly deteriorating exemplifies concern with stability; and assessment of patients through Emergency is related to the factor of time. An exception to safety and stability in which patients may be delegated to UNPs is when patients are placed on suicide precautions

Your organization has made a decision to implement TCAB in your hospital. As a manager, what strategies would you use to implement TCAB? (Select all that apply.) a. Encourage recognition among staff of their knowledge of the patient-care environment. b. With staff, select small changes for consideration. c. Select only projects that have widespread impact. d. Secure external advisors to evaluate innovation. e. Present ideas based on best practices and ask staff for advice on implementation.

ANS: A, B, E Transforming care at the bedside (TCAB) relies on active involvement of staff in the generation of innovative ideas to improve patient care. Staff are actively engaged in selecting innovation, planning, and evaluation of the innovations. Critical to practice changes, rapid cycle change is a process that encourages testing creative change on a small scale while determining potential impact.

20. You ask Evelyn, a new UNP, to check what is left in Mrs. N.'s inhaler when Evelyn makes visits to Mrs. N. and also to check whether Mrs. N. is receiving any positive effect from the medication. Evelyn reports for 3 weeks that Mrs. N. is using the inhaler and that there is enough medication left in the device. The day of her last visit to Mrs. N., Mrs. N. is admitted to the hospital in severe respiratory distress. When she is admitted, she tells the physician that she has not been using the inhaler for 4 weeks. Determination of Evelyn's educational preparation and certification is related to the concept of: a. accountability. b. authority. c. role performance. d. assignment.

ANS: B Authority refers to the right to do and may be designated by law, educational preparation, or job description.

8. During staff-development programs, staff nurses verbalize their frustration about their workloads and having to delegate so many tasks to others. One of the main reasons that delegation has emerged as an issue is because of: a. the amount of paperwork required to complete care. b. the complexity of care required by patients. c. earlier discharge practices. d. the numbers of other disciplines present on a given unit.

ANS: B Complexity of client care, a multilevel nursing model (registered nurses, mixed with LPNs/LVNs, and UNPs), and community-based care provide many challenges in determining the care required and outcomes desired and/or mandated, and in matching needs with various abilities and authority of regulated and unregulated healthcare providers. The nurse manager should ensure that staff is clinically competent and trained in their roles in patient safety.

1. A group of staff nurses is dissatisfied with the new ideas presented by the newly hired nurse manager. The staff wants to keep their old procedures, and they resist the changes. Conflict arises from: a. group decision-making options. b. perceptions of incompatibility. c. increases in group cohesiveness. d. debates, negotiations, and compromises.

ANS: B Conflict involves disagreement in values or beliefs within oneself or between people that causes harm or has the potential to cause harm. Conflict may result from the interaction of interdependent people who perceive incompatibility and the potential for interference.

A member of a patient's family calls the nurse manager of the palliative care unit to express concern that a member of the family, who died on the weekend, had requested analgesics from the RNs on duty. An RN came with the analgesic nearly 45 minutes later, just after the patient had died. The manager is aware that the unit was especially busy that weekend because many patients were seriously ill, staff had called in ill, and the staffing manager was unable to completely replace staff who were absent. The manager is deeply troubled that the family member had to die in pain because it violates what she knows should have been done. This manager is experiencing: a. Compromised agency. b. Moral distress. c. Moral sensitivity. d. Moral dilemma.

ANS: B Moral distress is experienced when nurses cannot provide what they perceive to be best for a given patient. Examples of moral distress include constraints caused by financial pressures, limited patient care resources, disagreements among family members regarding patient interventions, and/or limitations imposed by primary healthcare providers. nurse managers can best assist nurses experiencing moral distress by remembering that such distress may be lessened through adequate levels of knowledge regarding nursing ethics and its application, acknowledging that such distress does occur and serving as an advocate for nurses.

One means of ensuring that nurses floated to other patient care areas in healthcare organizations are qualified to work in those areas is: a. Employing additional staff to assist with orientation processes. b. Cross-educating staff members to other areas of the institution. c. Transferring patients to units where the staffing pattern is optimal. d. Orienting staff members to all patient care areas as part of their general orientation to the institution.

ANS: B Nurses should be floated to units as similar as possible to their own to decrease the potential for liability. Negotiating cross-training, a proactive approach to temporary staffing problems, reduces the potential for liability.

To reduce the incidence of falls in a skilled nursing unit, the nurse manager contacts the risk manager. Risk management is a process that attempts to identify potential hazards and: a. Compensate for previous injuries. b. Eliminate these risks before anyone else is harmed. c. Supersede the need for staff members to file incident reports. d. Discipline staff members who have been involved in previous incident reports.

ANS: B Risk management involves taking proactive steps to identify and eliminate risks and liability.

To engage your staff in awareness of their current practice and how it is affirmed or not by evidence, you plan a short series of learning presentations on evidence and use of heparin and saline to maintain IV patency. You meet with the educator to plan out the goals for each session with the overall purpose of increasing knowledge and awareness of staff in readiness to consider questions related to the IV practice. Staff nurses who gain information on current IV therapy practices are engaging in which phase of Rogers' decision-making process? a. Persuasion b. Knowledge c. Confirmation d. Decision

ANS: B Rogers' innovation-decision process involves five stages for change in individuals, the first of which is knowledge.

While walking past a patient's room, you overhear one of the RN staff telling a patient that the patient has no right to refuse chemotherapy treatment because the family and the doctor think the treatment is the best option for the patient. This patient is 40 years of age and alert. When you meet later to discuss what you heard with the RN, it is important to: a. Discuss how statute law enforces the right of the doctor, but not of families, to ensure that patients comply with recommended treatment plans. b. Discuss that statute law provides for patient autonomy and refusal of treatment. c. Remind the nurse to provide clearer explanations to aid in the patient's comprehension of the treatment and compliance. d. Acknowledge the nurse's role in ensuring that she does not fail in her duty of care for the patient.

ANS: B Statute law states that the patient must be given sufficient information, in terms he or she can reasonably be expected to comprehend, to make an informed choice. Inherent in the doctrine of informed consent is the right of the patient to informed refusal. Patients must clearly understand the possible consequences of their refusal.

16. Decision making is described by the nursing educator as the process one uses to: a. solve a problem. b. choose between alternatives. c. reflect on a certain situation. d. generate ideas.

ANS: B The hallmark of decision making is choosing among options. Generating options is one phase of decision making, and solving a problem refers to problem solving, which is problem centered. Decision making does not always begin with problems, but rather is defined as a purposeful, goal-directed effort that uses a systematic process to choose among options.

18. You ask Evelyn, a new UNP, to check what is left in Mrs. N.'s inhaler when Evelyn makes visits to Mrs. N. and also to check whether Mrs. N. is receiving any positive effect from the medication. Evelyn reports for 3 weeks that Mrs. N. is using the inhaler and that there is enough medication left in the device. The day of her last visit to Mrs. N., Mrs. N. is admitted to the hospital in severe respiratory distress. When she is admitted, she tells the physician that she has not been using the inhaler for 4 weeks. This incident is an example of: a. incompetence of the UNP. b. failure to follow-through. c. skills but no motivation. d. lack of accountability.

ANS: B The nurse should maintain open lines of communication and seek information, and the UNP should know how, when, and what to report. Communication of delegation of tasks includes specific information about what is being delegated, expected outcomes, and deviations (which includes what immediate action needs to be taken). This 2-way communication and follow-through allows patient care to be altered, if necessary, in a timely manner.

In a telehealth organization, a nurse who is licensed in New York and Pennsylvania provides teaching to a patient who resides in Pennsylvania. The patient charges that the teaching failed to provide significant information about a potential side effect, which led to delay in seeking treatment and untoward harm. Under which state nurse practice act and standards would this situation be considered? a. New York b. Pennsylvania c. Neither New York nor Pennsylvania d. Both New York and Pennsylvania

ANS: B Under the law, the state in which the patient resides and not the state where the nurse holds his or her license determines the state nurse practice act that is considered.

6. During a fire drill, the nurse manager becomes very assertive and directive in her communications with staff. This type of situational leadership depends on: a. supportive behavior by the leader and immature followers. b. the development level of the followers and the behavior based on the situation. c. well-developed followers combined with a strong leader who acts quickly. d. the leader's ability to evaluate personnel and communicate that evaluation.

ANS: B When abilities, relationships, and/or time is limited (as in a crisis situation), the leader assumes a bigger role in guiding and in making decisions, or "telling" behavior. Leaders need to behavior differently and use different leadership styles in different situations.

19. You ask Evelyn, a new UNP, to check what is left in Mrs. N.'s inhaler when Evelyn makesvisits to Mrs. N. and also to check whether Mrs. N. is receiving any positive effect from the medication. Evelyn reports for 3 weeks that Mrs. N. is using the inhaler and that there is enough medication left in the device. The day of her last visit to Mrs. N., Mrs. N. is admitted to the hospital in severe respiratory distress. When she is admitted, she tells the physician that she has not been using the inhaler for 4 weeks. Before assigning Evelyn to Mrs. N.'s care, the most appropriate action of the care coordinator would have been to: a. determine Evelyn's educational background and preparation for this role. b. ask Evelyn if she has worked with inhalers before and to describe what she knows about them. c. advise that if Evelyn has any questions about what to do with the inhaler, she should come to the coordinator. d. advise Evelyn that working the inhaler is not really complicated and that she should ask the patient how to check medication levels in the inhaler.

ANS: B When delegating tasks, in addition to specifying the task to be completed, outcomes, priorities, time lines, deviations, report time frames, monitoring, and resources, asking the delegatee to give examples of each is helpful in ensuring that communication is clear and has been understood. Preparation of UNPs lacks consistency; therefore, the safest practice is to determine the knowledge and skill level of the UNP in relation to the skill and the patient before delegating.

13. Which of the following indicates safe delegation? a. The nurse supervisor for a large urban acute care department asks the unit manager to accept two new acutely ill patients, which the manager does. The unit is short two staff, and the replacement is inexperienced. b. A unit manager agrees to release a staff from her unit to Unit B. The staff member she agrees to release is experienced on Unit B and is agreeable to the change. The unit manager's unit is fully staffed and patients are stable. c. The nurse supervisor asks the head nurse for Unit A to make do without a replacement for an ill staff member because Unit A was originally overstaffed anyway. Patient acuity levels are very high on Unit A and two staff are orientating. d. The nurse supervisor asks the charge nurse on Unit B to cover Unit F, which is two floors up, because the charge nurse for Unit F is ill. The charge nurse for Unit B is an experienced manager but has no experience with the nursing care required on Unit F.

ANS: B When span of control (number of individuals for whom a manager is responsible) is compromised by geographic factors such as lack of proximity, instability in patients' conditions, or lack of experience, the span of control that is being delegated may lead to unsafe care.

In a nurse managers' meeting, the chief nursing officer encourages the managers to brainstorm ways to reduce costs. Nurse managers have the greatest impact on reducing costs by managing: a. supplies. b. staffing. c. fixed costs. d. medication costs.

ANS: B Because staffing constitutes the largest portion of any healthcare budget, managing the mix and numbers of staff required for patient care to meet identified outcomes will have the largest impact on budgets.

The chief nursing office continues to seek ways to improve healthcare services to clients and to save the hospital money. However, with the federal guidelines of paying agencies based on capitation, the chief nursing office faces a challenge. Capitation provides incentives for healthcare providers to control costs by: a. providing fewer services to fewer clients. b. using fewer services per client. c. using high-technology treatments. d. requiring second opinions.

ANS: B In a capitated environment, a single fee is paid for all services provided. To be financially viable under this reimbursement model, organizations would be interested in decreasing the volume of services used and increasing the volume of patients. High-technology treatments and second opinions may increase the number of services used.

The chief nursing office of a Magnet® hospital has conducted a study of ways to improve healthcare services. Healthcare services that add value for clients: a. accomplish healthcare goals. b. minimize costs. c. decrease the number of services used. d. use high-technology treatments.

ANS: B Models of reimbursement affect which services and approaches (e.g., decreasing the number of services used) might be financially viable and add value for clients. It is critical to determine and advertise the value of nursing care. Services that add value are of high quality, affect health outcomes positively, and minimize costs.

In hiring nurses during the transition from team nursing to a primary nursing model, Benner's work would suggest that you give priority to nurses who are at least at which level of competency? a. Advanced beginner b. Competent c. Proficient d. Novice

ANS: B Nurses who have less than 2 or 3 years' experience in primary nursing and/or less than 2 or 3 years of nursing experience will likely require more assistance than other nurses, which will put a greater demand on the unit during a time of transition.

The nurse manager at a cardiac rehabilitation unit was asked to select a care delivery model. Which of the following methods would be the most cost-effective? a. Functional method b. Case management method c. Primary care method d. Team method

ANS: B Team nursing, functional nursing, and case management are all considered efficient, cost-effective methods of care delivery because they enable utilization of various types of healthcare providers (rather than baccalaureate nurses in direct care, which is the primary nursing method). Case management is considered particularly cost-effective in patient care settings because it maintains quality care while streamlining costs for high-risk, high-volume, high-cost patient populations and seeks the active involvement of the patient, the family, and diverse healthcare professionals.

When interviewing an applicant for an RN position, the nurse manager describes the unit's care delivery system as one in which the nursing assistants are cross-trained to perform specific tasks and the RNs complete all treatment, medication administration, and discharge teaching. The nurse applicant knows this nursing care delivery strategy to be: a. the case method. b. functional nursing. c. primary nursing. d. nurse case management.

ANS: B The functional model of nursing is a method of providing patient care by which each licensed and unlicensed staff member performs specific tasks for a large group of patients.

In transitioning to a primary nursing model, it is important for a nurse manager who enjoys a high level of control over patient care to understand that his or her decision making at the patient care level: a. is increased. b. is decreased. c. is relinquished. d. remains the same.

ANS: B The nurse manager who is considering movement to primary nursing needs to consider how the role of the manager changes, as well as the roles of the staff. The role of decision making at the patient care level is relinquished to the primary nurse, and the role of manager becomes that of facilitator, coach, mentor, role model, and clinical resource.

You are considering putting forward a proposal to move the model of care from team nursing to a primary nursing hybrid: patient-focused care model. In considering this proposal, you recognize that significant costs specific to operationalizing this model are related to: a. implementation of an all-RN staff complement. b. significant changes in the physical structure of units. c. orientation of staff to new roles and responsibilities. d. testing and piloting technology at the bedside.

ANS: B This particular model includes a focus on patient care that includes multidisciplinary teams and assistants at the bedside. Services, including laboratories and pharmacies, are decentralized to bring them closer to where care is delivered.

Within a healthcare environment, where the gap between revenues and costs can mean the difference between sustainability of an organization and nonsustainability of an organization or services, it is critical for nurse managers to: a. maintain a clear vision of how to trim healthcare costs. b. balance value-added services against costs and revenues. c. consistently delete programs that are of high cost. d. implement programs that bring in additional revenues.

ANS: B To achieve and maintain financial viability, nurse managers must be able to think strategically financially and in terms of nursing care. Cutting costs by deleting programs and bringing in additional revenue through new programs and services are not in themselves strategic unless the decisions made lead to quality care, have positive outcomes, and are efficient in terms of cost.

After reviewing her monthly budget report, the nurse manager sees that she has a negative variance, which prompts her to change the staffing schedule. A negative or unfavorable variance in a monthly expense report may result from: a. overestimation of inflation. b. higher than expected client acuity. c. net revenue exceeding net expenses. d. not replacing staff who called in sick.

ANS: B Variance reflects the difference between what was projected and the actual performance in a budget. When the variance is negative or unfavorable, the amount spent is more than what was budgeted (expenses exceed revenue); this may be a result of higher acuity. To help managers interpret and use variance information better, some institutions use flexible budgets that automatically account for census variances.

To develop a curriculum vitae, or résumé, a nurse must develop a personal data file. The goal of a personal data file is to: a. create an opportunity to be interviewed. b. have a listing of facts about your professional life. c. have a tool in place for marketing yourself. d. create a document that lists your skills.

ANS: B A personal data file enables storage and recall of career-specific details that can be retrieved and shaped for a specific purpose using cut-and-paste approaches rather than creating whole new documents.

A nurse staffing plan takes into account: a. specific nurse-to-patient ratios per shift. b. participation of nurses in projecting staffing needs. c. compensation and benefits for each level of staff. d. the occupancy load of a unit.

ANS: B Nurse staffing plans employ nursing judgment and flexibility that is based on acuity, nurse experience, and unit configuration rather than set nurse-to-patient ratios. The American Nurses Association has opted to support the nurse staffing committee as the approach to ensure safe staffing. In 2013, national legislation was introduced that requires all acute care hospitals to establish a committee made up of 55% direct care RNs.

To project staffing needs and to avoid understaffing, it is important that nurse managers consider which of the following? a. Maximum productive hours b. Average non-productive hours c. Minimum benefit hours d. Maximum vacation time

ANS: B To avoid understaffing, average non-productive or benefit hours need to be taken into account, so the unit is properly staffed when staff members are off.

A particular classification system assigns revenue according to the functional capacity of patients and the progression of patients during their stay in rehabilitation units. More independent patient activities, such as prompted voiding, require higher staff utilization than dependent activities but do not result in increased staff resources. This is an example of: a. bureaucracy. b. concern related to the validity of classification systems. c. inadequate reliability of classification systems. d. inappropriate subjectivity in making judgments about staffing.

ANS: B Validity of categories and implications for staffing levels are in question in this situation because staffing levels are not reflective of the levels of activity required for patient care.

You volunteer at a free community clinic. A 13-year-old girl claims to have been diagnosed with SLE and presents with chlamydia. The team leader at the clinic advises that: a. The state-defined age of legal consent is 18; therefore, no treatment can be delivered. b. The teen is underage and should be referred to the family general practitioner. c. Care can be provided as long as consent is voluntary and information about treatment and options is provided. d. Treatment is provided as long as telephone consent is obtained from a parent or legal guardian.

ANS: C All states have a legal age for consent; generally, this age is 18. However, emancipated minors, minors seeking treatment for substance abuse, and minors seeking treatment for communicable diseases can provide their own consent.

11. Mrs. Hill, aged 68, was hospitalized after a stroke. The speech therapist recommended that oral feeding be stopped because of her dysplasia. During visiting hours, Mr. Hill fed his wife some noodles. The nurse noticed this and stopped Mr. Hill from feeding his wife, telling him it was the doctor's decision. An hour later, the nurse returned and found Mr. Hill feeding his wife again. The nurse tried to stop him again. Mr. Hill refused and claimed that the clinical staff was trying to starve his wife; he also threatened to get violent with the nurse. The nurse decided to walk away and documented the event in Mrs. Hill's chart. According to Thomas' four stages of conflict, in which stage could the nurse have been more effective? a. Frustration b. Conceptualizing c. Action d. Outcomes

ANS: C By walking away, the nurse is engaged in an action or a behavioral response, which is the action stage of conflict that is outlined in the four stages of conflict (Thomas, 1992). In this stage, the nurse might have used more effective strategies, such as clarifying Mr. Hill's views on feeding his wife and engaging in dialogue with Mr. Hill to clarify his concerns and attempt to reach a common goal.

8. A nursing instructor is teaching a class on conflict and conflict resolution. She relates to the class that conflict in an organization is important, and that an optimal level of conflict will generate: a. creativity, a problem-solving atmosphere, a weak team spirit, and motivation of its workers. b. creativity, a staid atmosphere, a weak team spirit, and motivation of its workers. c. creativity, a problem-solving atmosphere, a strong team spirit, and motivation for its workers. d. a bureaucratic atmosphere, a strong team spirit, and motivation for its workers.

ANS: C Differences in ideas, perceptions, and approaches, when managed well, can lead to creative solutions and deepened human relationships. Work on conflict suggests that complete resolution of conflict is counterproductive to the achievement of organizational goals, organizational change, and cohesiveness of employees.

16. An RN colleague, who is a long-standing and collaborative member of your team, is performing a complex dressing with new orders written for the first time for the assigned patient. Which of the following would be the most appropriate communication with her? a. "How do you usually do this kind of dressing?" b. "The dressing needs to be done today and tomorrow with the supplies on this cart." c. "Here is what you need for the dressing, and I will show you what needs to be done." d. "I know you know what you are doing. Let me know if you have any problems."

ANS: C If a situation involves a new task and the relationship is ongoing (two individuals who will usually continue to work together), the delegator explains what to do and how to do it. Hersey described the leader's behavior as explaining or persuading, which, is characterized as "selling." The RN who is assigned to the patient is an experienced nurse and team member, but is new to this specific situation. In situations where the nurse is experienced but the task is new, explain (and demonstrate) what needs to be done.

7. The unit manager is working in a large metropolitan facility and is told that two UNPs are to be assigned to work with her. Delegation begins with: a. acknowledging the arrival of the second UNP on the unit. b. providing clear directions to both UNPs. c. matching tasks with qualified persons. d. receiving reports from the prior shift.

ANS: C In delegating to the UNPs, the nurse must consider what cannot be delegated, as well as the factors of safety, time, critical thinking, and stability of patients.

The clinical coordinator expects the position description of the new wound care specialist to change nurses' responsibilities in caring for clients with skin integrity problems. The best approach to address this need for change, yet to have the best outcomes for clients, staff nurses, and the organization, is to: a. Select one of the change models. b. Use Lewin's model and principles of change. c. Apply both planned and complexity theory approaches. d. Form a task force of nursing staff and wound care specialists.

ANS: C In the second stage, the moving or changing stage of Lewin's theory, planned interventions and strategies, such as education, vision building, and incremental steps towards the change, are executed to support the implementation of the change. This situation potentially also involves complexity theories that recognize that change involves engagement of individuals and subsystems throughout the unit and organization.

As the unit manager on the unit that is leading changes to heparin locks, you find that Elizabeth is very valuable in terms of her observations about other units and her knowledge of organizational processes, and now in discussing the new procedure with others. Elizabeth might be considered an: a. Engager. b. Innovator. c. Informal change agent. d. Informant.

ANS: C Informal change agents are those who do not have formal, positional power but who have credibility through expertise and can model the new way of thinking, or who offer suggestions, ideas, and concerns

15. From the information supplied in this chapter, which statement best defines critical thinking? Critical thinking is a: a. high-level cognitive process. b. process that helps to develop reflective criticism for the purpose of reaching a conclusion. c. high-level cognitive process that includes creativity, problem solving, and decision making. d. discussion that guides the nursing process.

ANS: C It is generally accepted by many authors and researchers that this statement best defines critical thinking.

21. High-quality decisions are most likely to be made in nursing situations when: a. team leaders make the crucial decisions. b. individuals are advised of the problems. c. group size is neither too small nor too large. d. members are passively involved.

ANS: C Research has shown that group size is important. Too small a group means a limited number of options generated. Too large a group can mean lack of structure or lack of meaningful discussion.

In keeping with standards of The Joint Commission (TJC), the nurse manager organizes an orientation for new staff members. As part of the orientation, the nurse manager reviews the employee handbook. Employers may be bound to statements in the employee handbook: a. Under the doctrine of apparent agency. b. Under the doctrine of respondeat agency. c. Based on the employee's or the employer's expectations. d. Based on the theory that the handbook creates an explicit contract.

ANS: C The handbook is an implied contract and frames the employment contract.

23. County Hospital has position descriptions for all staff, including RN Team Leaders. Sarah, a team leader on the rehab unit, assesses the needs of the patients in her area, assesses the skills and backgrounds of each of the individuals on her team, and then assigns and delegates the appropriate care provider to each patient and task. Sarah provides Colleen, her RN colleague with details regarding the patients to whom Colleen has been assigned on the day shift. This is an example of: a. accountability. b. responsibility. c. assignment. d. delegation.

ANS: C When an RN assigns care to another RN, it is termed an assignment and not delegation, because both accountability and responsibility are transferred.

The principle that requires nurses to uphold a professional code of ethics, to practice within the code of ethics, and to remain competent is which of the following? a. Veracity b. Autonomy c. Fidelity d. Honesty

ANS: C ethical principles: Autonomy, Beneficence, Nonmaleficence, Veracity, Justice, Paternalism, Fidelity, Respect for others. Fidelity refers to promise keeping or upholding one's promise to practice as a reasonable and prudent nurse would do and in an ethically competent manner. Veracity: telling the truth and demands that the truth be told completely. autonomy: address personal freedom and self-determination, the right to choose what will happen to oneself as well as the accountability for making individual choices.

The mediator asked each staff member to reflect on his or her communication style. Which of the following best describes communication? Communication: a. is a reflection of self-analysis. b. is a result of thoughtful consideration. c. consists of thoughts, ideas, opinions, emotions, and feelings. d. focuses on the sender of the message.

ANS: C Communication involves both senders and receivers and may or may not be a reflection of self-analysis and thoughtful consideration. It always, however, involves thoughts, ideas, opinions, emotions, and feelings.

An older adult couple with limited means and on Medicare is considering options after the hospitalization of Mrs. A. with a fractured hip. Mrs. A. is stable but requires assistance with bathing, transfer, and mobility, and this will present stress for Mr. A., who was hospitalized with a mild myocardial infarction last year. Considering their means and health concerns, which of the following might be the best option? a. Hospice care b. Custodial nursing home care c. Home care d. Hospital care

ANS: C Custodial nursing care is not covered under Medicare and therefore, the financial burden of this option may cause further stress for the couple. Medicare Part A is an insurance plan for hospital, hospice, home health, and skilled nursing care that is paid for through Social Security. Because Mrs. A. is stable and not terminal, she does not require hospice care. The assistance provided through home care is covered under Medicare and provides assistance for needs such as those of Mrs. A.

In preparing her budget, a nurse manager determines that she needs to budget for six FTE RN positions in the upcoming year. Based on a 40-hour week, this means that the nurse manager has determined that the budget will provide for _____ hours. a. 12,480 productive b. 10,820 productive c. 12,480 paid d. 10,800 patient care

ANS: C From the information given, it is not possible to determine how many non-productive hours (vacation, holiday, sick time, education) the nurse manager has accounted for in her budget calculations and therefore whether the total number of hours (40 hours 52 weeks 6 staff = 12,480 paid hours) is productive (paid time that is worked) or non-productive hours. 12,480 accurately reflects the total number of hours of work paid per year.

A patient complains to the charge nurse that she has no idea who "her nurse" is on any given day. "I ask one nurse for my pills and she says, 'That's not my job.' I ask the pill nurse about my lab tests and she says that I should ask another nurse." The nursing care delivery model most likely employed in this situation is: a. differentiated practice. b. team nursing. c. functional nursing. d. case management.

ANS: C Functional team nursing involves licensed and unlicensed personnel who perform specific tasks for a large number of patients. A disadvantage of functional team nursing is the fragmentation of care. The physical and technical aspects of care may be met, but the psychological and spiritual needs may be overlooked. Patients become confused with so many different care providers per shift. These different staff members may be so busy with their assigned tasks that they may not have time to communicate with each other about the patient's progress.

In considering whether or not to accept a job offer as a nurse manager at a Magnet® hospital, you look at an environment that you might encounter as a head nurse at the hospital. You determine that you could expect to: a. find it difficult to recruit new staff. b. see rapid turnover of staff on your unit. c. find nurses who exemplify interest in quality care. d. find limited interest in excellence in the nursing environment.

ANS: C Magnet® hospitals exemplify hospitals whose focus processes attract and retain nurses who value and promote quality care and excellence in nursing environments.

A nurse manager questions the true difference between primary nursing and total patient care. After careful consideration of both models, the nurse manager concludes that primary nursing differs significantly from total patient care in: a. breadth of nursing knowledge and expertise required. b. intention to provide holistic nursing. c. degree of task orientation. d. levels and types of assessment.

ANS: C Significant overlap can be seen between primary nursing and total patient care in terms of breadth of assessment and knowledge required to provide holistic care. A primary difference is that nurses in the total care model assume accountability while on shift, whereas primary nurses assume responsibility from time of admission to discharge and 24 hours a day.

A nurse manager approves two staff nurses to attend a national conference. When reviewing the budget, the nurse manager looks at which line item? a. Cash budget b. Capital budget c. Operating budget d. Supply and expense budget

ANS: C The operating budget includes a personnel budget, which takes into account productive and non-productive paid work hours. Education hours are covered under nonproductive paid work hours in the operating budget.

In a small rural nursing home, a director of nursing decides, because of a shortage of nurses, to implement a partnership model to help with basic tasks that comply with state rules regarding delegation. What type of design constitutes a partnership care delivery model? a. RN and LPN/LVN b. RN and RN c. RN and medication assistants d. RN and certified nurses' aides

ANS: C The partnership care delivery model is a variation of primary nursing in which an RN works with a consistent assistant, who performs basic nursing functions consistent with state delegation rules.

The case method of care delivery could be best justified in which of the following scenarios? a. Stable patient population with long-term care and family needs b. Acute care surgical unit with predictable postsurgical outcomes and many technical procedures c. Pediatric intensive care unit that heavily involves families as well as patients d. Home healthcare environment with patients at varying levels of acuity

ANS: C This model is especially useful in the care of complex patients who need active symptom management provided by an RN, such as the care of the patient in a hospice setting or an intensive care unit. This method would be justifiable delivery in the pediatric intensive care unit, where the status of patients can change rapidly and where complex functions of care involve both patients and families.

A well-written letter of resignation is critical to: a. first announce your intent to resign. b. formally signal discontent in your current position. c. maintain a positive relationship with your former organization and colleagues. d. fulfill your legal obligations as a departing employee.

ANS: C A well-written resignation letter outlines your intent to leave the organization and your appreciation of the organization but should follow an initial meeting with your manager to first discuss your intention. A well-written letter maintains a positive relationship with the organization.

A unit manager recently graduated as a clinical nurse specialist with a focus in gerontology. She applied to take a certification exam. Certification is designed to recognize: a. basic knowledge in a specified area. b. advanced practice in functional roles. c. special knowledge beyond basic licensure. d. continued competence as a registered nurse.

ANS: C Certification is an expectation in some settings for career advancement in advanced practice or in specialized areas and goes beyond basic preparation.

During an interview for a manager's position, you find the supervisor and staff unfriendly. Responses to questions are met with vague responses. After the interview, you decide not to pursue the position. What follow-up, if any, is most appropriate? a. There is no need for you to do anything further. You likely will not be offered the position anyway. b. You should file a complaint with human resources about the supervisor's lack of interviewing skills. c. You should send a thank-you note to the interviewer, indicating appreciation for her time. d. You should call and leave a voicemail, indicating your disinterest in the position.

ANS: C Even if you are disinterested in the position or think that the interview has gone badly, an appropriate follow-up is a thank-you note to the interviewer. This recommended follow-up creates a positive impression and may leave open the possibility of future interactions.

During performance appraisal interviews, Joanne's nurse manager notices Joanne's excitement when she talks about how she has helped patients on a rehab unit understand the complexities of their regimens. When Joanne's nurse manager asks her about her career path plans, Joanne says that she wants to become a nurse administrator. The best response to Joanne would be: a. "Nursing administration is rewarding. What experiences would help you along this path?" b. "You do not appear excited about nursing administration. Unless you are excited by that career path, I wouldn't advise going in that direction." c. "You seem to find teaching others very rewarding. Have you considered that as a possible career path?" d. "You are too inexperienced to consider administration. Work for a few years, and then consider administration."

ANS: C Joanne evidences excitement about teaching patients, and although administration could be a rewarding path for her, education might be a better option for her to consider. Looking at job aspects that are rewarding is helpful in determining which career direction to pursue.

In a job interview for a staff position, which of the following indicates your knowledge of patient safety? a. "Will I be able to get overtime hours on your unit?" b. "If there is an opportunity to work extra shifts, I would really like that." c. "Is there a strategy in place to reduce the number of overtime hours on the unit?" d. "I see no reason why I wouldn't be able to work overtime."

ANS: C Overtime, whether voluntary or mandatory, to fill staff vacancies is seen as a risk to both patients and nurses because it is more likely to lead to compromised decision making and technical skills because of fatigue.

In evaluating weekend mortality rates, the head nurse on the cardiac unit is surprised to find that they are higher than on weekdays. In exploring the reasons for this apparent anomaly, the head nurse focuses on: a. availability of diagnostic personnel. b. availability of physicians. c. communication with on-call providers. d. acuity level of patients.

ANS: C Studies to date of off-peak hours (weekends and nights) are limited, but those that have been done indicate increased mortality during weekends and nights, when staff work with fewer and often less experienced staff and when there may be strained communication among on-call healthcare providers.

Which of the following situations is most likely to result in a productive, whole work situation? a. Amy, RN, 5 years of experience in the Emergency Department. Amy accepts a position working with older clients in a home health agency because she has relocated, and this is the only full-time position available. b. Adam, RN, 8 years of experience in various nursing positions, including that of a nurse manager. Adam accepts a new nurse manager position because he has a family and wants more regular hours. He is most comfortable working in direct client care. c. Louise, RN, 10 years of experience in an Emergency Department. She accepts a position as a case manager in home health care, working with older clients. She especially enjoys working with older adults and wants to take on leadership and management challenges. d. Courtney, RN, a new graduate. Courtney is getting comfortable with delivering nursing care as an RN. She is offered a position on surgery as a permanent team leader. The unit has had a great deal of turnover recently, and only limited mentorship is available.

ANS: C The whole of any work situation is composed of two elements: person and position. A productive, whole work situation results when a person's talents and strengths are successfully blended with expectations of the position. Of the situations described previously, the one most likely to result in a productive, whole work situation is that of Louise, who, although her experience has not been in home health, is interested in both the roles and the responsibilities of the position, as well as the target group being served. She also considers the group being served as an area in which she demonstrates strength.

An example of a career is: (Select all that apply.) a. employment in short-term contract jobs in business, nursing, and whatever is available. b. involvement in an area of practice that is regulated. c. continuous employment in the same position and the same arrangement for 20 years. d. moving into and out of nursing positions in various cities while pursuing travel and education that develop understanding of global health.

ANS: C, D Career refers to progression of skills, consistency, knowledge, and/or status. This movement through nursing life is predicated on having a vision of a career as opposed to a series of jobs. Career styles can be defined as linear, steady state, entrepreneurial, or spiral. Deepening skills in one area is an example of a steady state career style, whereas moving into and out of positions in various cities can characterize an entrepreneurial style. Involvement in a regulated field defines a professional interest but not necessarily a career.

During a staff shortage, you hire an RN from a temporary agency. The RN administers a wrong IV medication that results in cardiac arrest and a difficult recovery for the patient. Liability in this situation: a. Is limited to the temporary agency. b. Is restricted to the RN. c. Could include the RN, the agency, and your institution. d. May depend on the patient's belief regarding the employment relationship.

ANS: D Apparent agency may apply here because your liability and that of your institution could be established if it can be shown that the patient believes that the RN was an employee of yours and of your institution. Apparent agent/apparent authority refers to the doctrine whereby a principal becomes accountable for the actions of his or her agent, it is created when the person (agent) holds himself or herself out as acting in behalf of the principle, in the instance of the agency nurse, the patient cannot ascertain where the nurse works directly for the hospital or is working for different employer. this principle applies when it can be shown that a reasonable patient believed that the healthcare worker was an employee of the institution.

23. What exemplifies the predominant style of conflict management for staff nurses? a. Sarah and Jonas, two RNs, disagree about the best approach to assisting a family that has complex needs. They decide that they will consult with family and together will decide what is best. b. Jennifer needs to switch a shift to attend a family function. She arranges to trade with Nancy, who wants a day off next to a 3-day break. c. Lindsay asks Melody to stay late for the third day in a row. Melody refuses, stating that she has already helped out for 2 days by staying late for Lindsay. d. Lara asks Stacey to switch shifts with her because Lara wants to attend a concert. Stacey would prefer not to but does to enable Lara, who is new in town, to be with her friends.

ANS: D Avoidance and accommodation are the predominant conflict management styles of nurses. Accommodating involves neglecting one's own needs while trying to satisfy the needs of another.

20. The head nurse and a staff nurse are having a conflict over how to use and apply a new procedure for dressings in the medical/surgical area. The staff nurse wishes to use the new procedure based on newly released nursing research. The head nurse wishes to use a protocol that has been used in the department for a number of years. The head nurse later makes comments to other staff on her unit about the credibility of the staff nurse. This behavior is associated with: a. lateral violence. b. horizontal violence. c. confrontation. d. bullying.

ANS: D Bullying involves aggressive or destructive behavior or psychological harassment of a recipient who is in a position of power differential with the perpetrator (the head nurse). Bullying is closely related to lateral or horizontal violence and involves such behaviors as incivility or intimidation.

21. Which of the following exemplifies the predominant conflict management style of nurse managers? a. Elizabeth, the head nurse on neurology, finds that Tom, the RN nurse on nights, is irritable in relation to any suggestions or new ideas, and so she comes in to work after Tom leaves the unit. b. The technology committee has recommended a clinical system for implementation on the nursing unit. Staff is anxious about the change. Tim, the head nurse, asks staff for ideas on how to meet the technology goals and to meet staff needs. c. During management meetings, George, the head nurse on nephrology, dominates meetings and decisions. Lee, the head nurse on the cardiac step-down unit, begins to miss the management meetings. d. Ann, RN, asks her head nurse if she can go on the permanent evening shift. The head nurse, Rajib, agrees, as long as Ann agrees to be involved in assisting to mentor evening staff in the use of the new clinical information system.

ANS: D Compromise involves trading and negotiation and is the predominant conflict management style of managers.

19. Factors that influence the ease with which conflict is resolved include all except which of the following? a. Level of interdependence of the parties b. Interprofessional collaboration c. Expression of one's own needs and ideas d. Avoidance of the issue or concern

ANS: D Conflict involves a level of interdependence and is a condition for conflict but not necessarily for continuance of the conflict. Expression of one's ideas and concerns is considered assertive and effective in resolving conflict if the concerns and needs of the other are also considered. Interprofessional collaboration has been shown to be effective in resolving conflict. Avoidance tends to prolong and sometimes escalate conflict.

15. In delegating to a UNP in a home health setting, which of the following represents the most appropriate delegation communication? a. "You will be taking care of Mrs. S., who needs assistance with her bath." b. "You will need to help Mrs. S. get into and out of her shower. Ensure that you check the condition of her feet, and let me know if you have any concerns when you check in." c. "I am not sure that you know how to do this, but I am giving you Mrs. S. She is quite obese and needs skin care." d. "Mrs. S. needs help to get into and out of her bathtub. Her bath will need to be completed by 10:00. When you are helping her to dry, please check between her toes and toenails, and phone me by 10:30 if you notice nail discoloration or redness."

ANS: D Delegation communication includes what is being delegated (and what is not), outcomes, specific deadlines (if applicable), specific reporting guidelines (what, when), and who may be consulted. Communication also includes conveying recognition of the authority to do what is expected.

5. The night nurse understands that certain factors need to be considered before delegating tasks to others. These factors include the: a. complexity of the task and the age of the delegatee. b. potential for benefit and the complexity of the task. c. potential for benefit and the number of staff. d. complexity of the task and the potential for harm.

ANS: D In delegating tasks to others, the nurse considers factors such as stability of the patient, safety of the situation and of the patient, time and intensity involved, and level of critical thinking required to achieve desired outcomes.

Sarah, RN, is one of your most enthusiastic staff members and has been to a workshop on preparing educational materials for patients. On the basis of this workshop, she would like to develop an information Website for patients who are being admitted to the ward. An appropriate response to Sarah's suggestion would be: a. "That is a great suggestion, but we have no resources for such an expensive undertaking right now." b. "Perhaps you can keep that in mind as we redesign our charting system." c. "We have too many seniors as patients, and you know that they don't use technology." d. "There is a great group here that meets to look at technology pilots. Let's see if you can join them and discuss your idea further."

ANS: D Involving Sarah with others who enjoy new ideas and who are able to try out new ideas in pilot projects enables her to remain on the cutting edge and to try out innovative solutions with the least amount of disruption.

4. The nurse manager decides to use a mediator to help resolve the staff's conflict. A basic strategy for truly addressing this conflict is to: a. identify the conflicting facts. b. be determined to resolve the conflict. c. schedule a meeting time for resolution. d. have a clear understanding of the differences between the parties in conflict.

ANS: D It is important for each person in the conflict to clarify the conflict as "I see it" and how "it makes me respond" before all the persons involved in the conflict can define the conflict, develop a shared conceptualization, and resolve their differences.

The manager in the coronary care unit believes that the most important ethical considerations in performance evaluations are that they include the employee's good qualities and that they give positive direction for professional growth. This belief is an example of: a. Justice. b. Fidelity. c. Beneficence. d. Nonmaleficence.

ANS: D Nonmaleficence refers to "doing no harm." For a nurse manager following this principle, performance evaluation should emphasize an employee's good qualities and give positive direction for growth. Destroying the employee's self-esteem and self-worth would be considered doing harm under this principle.

24. In trying to achieve Magnet® status, the chief nursing officer establishes a shared governance model to help nurses experience job satisfaction. However, some nurses who have enjoyed working with less autonomy resist this change and begin to criticize and make rude comments about managers who embrace this model, as well as colleagues who support it. The comments are largely ignored because those who are making them are well established nurses who are often vocal about their displeasure with the organization. Organizational conflict isarising from which of the following? a. Staffing practices b. Increased participation in decision making c. Allocation of resources d. Tolerance of incivility

ANS: D Organizational conflict arises from discord related to policies and procedures (such as staffing policies and practices and allocation of resources), personnel codes or conduct or accepted norms of behavior (such as incivility), and patterns of communication. A major source conflict in organizations stems from strategies that promote more participation and autonomy of staff nurses.

12. Mrs. Hill, aged 68, was hospitalized after a stroke. The speech therapist recommended that oral feeding be stopped because of her dysplasia. During visiting hours, Mr. Hill fed his wife some noodles. The nurse noticed this and stopped Mr. Hill from feeding his wife, telling him it was the doctor's decision. An hour later, the nurse returned and found Mr. Hill feeding his wife again. The nurse tried to stop him again. Mr. Hill refused and claimed that the clinical staff was trying to starve his wife; he also threatened to get violent with the nurse. The nurse decided to walk away and documented the event in Mrs. Hill's chart. The outcome as depicted by Thomas' conflict stages can be considered to be: a. compromising. b. confronting. c. constructive. d. destructive.

ANS: D Resolution was absent because the nurse did not have time to effectively deal with the issues in the conflict. This can lead to negativity, increased frustration, and further distancing between individuals or groups, including between patients and nurses.

17. Sally is an experienced nurse on the unit and is very experienced with ICP monitoring. She is assigned David, a patient who has been admitted with a severe head injury. In communicating with Sally, what does is an appropriate action by the charge nurse? a. Provide a detailed explanation of what she needs to do with ICP monitoring. b. Tell her when she needs to provide an update about David's status. c. Ask her to tell you what she knows about ICP monitoring and share expectations about reporting. d. Advise her that you are available if she needs you.

ANS: D The charge nurse and Sally have a well-established relationship and Sally has the expertise towork effectively with David; therefore, the charge nurse would need to provide little guidance but would need to communicate that they are available if needed. Hersey refers to this leader behavior as "delegating."

3. A key advantage that a charge nurse has in terms of delegating is that: a. clients receive less attention because too many staff make it difficult to coordinate care. b. nurses report less pressure to perform necessary tasks themselves. c. administration can predict overtime more accurately. d. team skills can be used more effectively.

ANS: D The use of multilevel healthcare providers enables healthcare organizations and nursing to provide patient-centered care, with a focus on abilities and skills that can be employed to perform "what is needed now." As tasks become more complicated, delegating skills to others enables the nurse to effectively deliver a complex level of care.

10. You are working in a home health service and have three unlicensed nursing personnel (UNPs) assigned to your team. You have worked with two of them for 2 years; the third is new. The two experienced UNPs have patients with complex illnesses for whom they provide basic care. The third member of the team has been assigned to patients with less complex illnesses. Your best approach to supervising their care is to: a. remain in the office and ask each UNP to check in with you upon arrival at their first patient care site. b. ask another RN to supervise the two experienced assistants so you can be with the new person full time. c. meet the new staff member at the first patient care site and ask the others to call if anything is unusual. d. meet the new staff member at the first patient care site and call the others with questions to determine whether anything is unusual.

ANS: D When ability and willingness are strong, the involvement of the delegator is needed less.

The nurse manager is comparing functional nursing and primary nursing for potential adoption. The nurse manager determines that patient and nurse satisfaction in primary nursing are: a. similar to those in functional nursing. b. not of significance in either model. c. low by comparison with functional nursing. d. high when compared with functional nursing.

ANS: D Although some studies suggest there is little difference between functional nursing and primary nursing, primary nursing tends to be more satisfying for RNs, who enjoy a high level of accountability and autonomy in decision making. Patient satisfaction is also high, as patients form close therapeutic relationships with the nurse because of the continuity of the relationship. Functional nursing is criticized for low patient satisfaction and potential staff dissatisfaction related to boredom and an autocratic approach to management.

The unit manager was addressing nursing students in the lounge area and was discussing team leadership and team effectiveness. She stated, "One can agree to disagree with another team member's perspective even when one doesn't necessarily see that perspective as being the correct one." In being creative, what did she mean? a. Championing one's own opinion b. Being compassionate c. Being flexible d. Committing to resolution

ANS: D Caregivers must listen to the other person's perspective, listen to the message accurately, identify differences, and creatively seek resolutions.

As the nurse manager on a rehab unit, you are asked to come to the tub room immediately because two nursing assistants are having a loud disagreement in front of a patient. You ask the nursing assistants to meet you outside and after ensuring that a third nursing assistant is able to care for the patient, you speak with the two nursing assistants. Which of the following would you ask first? a. "How long have you two been working together?" b. "Have you experienced disagreements like this before?" c. "How do you think this patient's perception of her care has been changed?" d. "What happened to bring on this disagreement today?"

ANS: D Conflicts are usually based on attempts to protect a person's self-esteem or to alter perceived inequities in power. When a nurse recognizes upset and reaction, the following steps can be helpful (Sportsman, 2005): Identify the triggering event ("What happened to bring on this disagreement today?"). Discover the historical context for each person. Assess how interdependent each person is on the other. Identify the issues, goals, and resources involved in the situation

In the cardiac intensive care unit, there has been simmering discontent about the new nurse manager, who avoids any discussion about her scheduling and practice decisions. The staff have begun to sort into "different camps" depending on how they feel about the manager or the decisions. Which of the following statements MOST accurately describes this situation? a. The tension that has been generated will result in creative solutions. b. Staff will become a cohesive group that takes a stand against the manager. c. The conflict will result in increased dialogue about practice and scheduling options. d. Patient care may suffer because attention and energy is being diverted toward the unit relationships

ANS: D Destructive conflict polarizes groups, saps group morale, deepens differences in values, and diverts energy from more important activities, such as patient care. Constructive conflict opens up issues of importance, results in solutions to problems, and enables authentic communication.

Complex care of acutely ill patients is required on a surgical unit, which utilizes differentiated nursing practice as its model of care delivery. On what is the concept of differentiated nursing practice is based? a. Licensure status b. Experience in the agency c. Leadership capabilities d. Education and expertise

ANS: D Differentiated nursing practice models are models of clinical nursing practice that are defined or differentiated by level of education, expected clinical skills or competencies, job descriptions, pay scales, and participation in decision making.

In an acute care unit, the nurse manager utilizes the functional nursing method as the care delivery model. The nurse manager's main responsibility is the needs of the: a. department. b. unit. c. staff. d. patient.

ANS: D In a functional nursing model, where other team members are focused on performing specific tasks, the nurse manager assumes primary responsibility for patient outcomes.

In preparing her budget, a nurse manager determines that she needs to budget for six FTE RN positions in the upcoming year. The nurse manager determines that RN 1 has 5 weeks of vacation and 3 days of education and has averaged 3 sick days a year. RN 2 has 6 weeks of vacation, has asked to attend a 3-day conference and a 2-day workshop, and has no history of sick time. RN 3 is new, and you anticipate 3 weeks of orientation, as well as 4 weeks of vacation. RN 4 has 6 weeks of vacation and has an ongoing health condition, so you anticipate 2 weeks of sick time, as well as 3 days for education. RN 5 has 5 weeks of vacation, has no education planned, and averages 5 sick days per year. RN 6 has 6 weeks of vacation, has no educational opportunities planned, and has been off ill for 4 weeks. How many hours of productive paid time can be anticipated for the budget? a. 12,480 b. 11,200 c. 10,848 d. 10,688

ANS: D Items that have a useful life of longer than a year, which is likely with the pumps, and that have a cost that usually exceeds $300 to $1000 (specific amount is set by the organization) are considered capital items. Operating budget items include what is used on a day-to-day basis, such as staffing.

The chief nursing officer works with her nurse managers by helping them understand how to develop and implement a budget. A nurse manager can best describe a budget as a: a. day-to-day plan for operations. b. unit of service. c. statement of revenues and services. d. financial plan.

ANS: D The budget is an overall financial plan that reflects organizational assumptions, objectives, and standards, and various types of budget planning, including operational and capital budgets, which, in turn, reflect revenues and costs.

The nurse case manager is working with a client admitted for end-stage renal disease. The case manager's major goal during this hospitalization for this patient is to: a. implement the care pathway on admission. b. provide direct nursing care throughout the hospitalization. c. supervise the nursing staff members who implement the care map. d. prevent additional hospitalizations resulting from complications of the client's disease.

ANS: D The goals and outcomes established in a critical pathway are designed to support the aims of case management, which are shortened hospital stays and prevention of hospital readmissions.

"I really wish that my supervisor would realize and acknowledge all the things I do well." In nursing, this has been identified as a problem. Which statement is part of the solution? Focus on: a. new staff. b. care assignments with which the individual is not familiar. c. making corrections. d. the strengths of the individual rather than the weaknesses.

ANS: D The research of Rath (2007) included many recommendations, one of which was that focusing on mediocre behaviors and on a person's weaknesses will not lead to excellence. Focusing on weaknesses tends to decrease the appreciation, and thus the acknowledgements.

Which of the following factors is not implicated in rising healthcare costs? a. Rising expectations of consumers for cure and care b. Marketing of drugs to consumers c. Large administrative staffs to process medical billings d. Rising Medicare costs

ANS: D Unintentional injuries, socioeconomic conditions (e.g., poverty and violence), marketing of pharmaceuticals, and the rising expectations of consumers with regard to what should be done to manage health concerns all contribute to rising healthcare costs. The costs of Medicare are not considered in relation to rising costs of health care.

You are offered an opportunity to take a temporary leave from your position as a nurse manager to lead a technology implementation project. Which of the following reasons for accepting the opportunity is most consistent with developing a solid career path? a. You are pressured to do so by your supervisor. b. The organization has no other suitable candidate for the position. c. You have limited knowledge of information technology and no real interest, but this will increase your knowledge. d. Accepting a position outside of your established skill set may establish you organizationally as an innovative, adaptable leader.

ANS: D Although giving into organizational or supervisory pressure may bring an enhanced learning and organizational profile, what is to be gained needs to be assessed against your career goals, interest, and aspirations. Increasing and expanding your skill set within defined career interests is a valid reason to consider a chance opportunity.

In preparing for a fair interview process as a hiring manager, you should: a. put water out for the candidates. b. ensure that you know the names of all candidates. c. dress comfortably and professionally. d. prepare a schedule of questions to be asked of all candidates.

ANS: D Although providing water, knowing names, and dressing appropriately sets a professional and respectful tone for the interview, developing a schedule of questions to be asked of all candidates is important for gathering comparative data and ensuring equitable treatment.

Your healthcare organization has a decentralized system for scheduling. As part of this process, after you have developed a draft schedule, you may need to: a. seek budgetary approval. b. balance personal schedules against institutional needs. c. negotiate the schedule with unit staff. d. submit the schedule to a centralized staffing office for review.

ANS: D In a decentralized model, you may be completely responsible for approving all schedule changes and for development of the schedule, or you may need to submit a draft to a centralized office for review and determination of supplemental staff. Balancing personal schedules and negotiation describes staff self-scheduling models.

A strategy to increase RN staff retention at Valley Hospital includes: a. better compensation and benefits. b. clearer position descriptions. c. lay-offs of nursing assistants. d. adequate staffing to meet acuity levels.

ANS: D Over the past decade, a significant amount of research has been done in the United States to evaluate links among nursing staffing, workloads, skills mix, and patient outcomes. An analysis of this research demonstrates that ensuring adequate staffing levels has been shown, among other things, to improve nurse retention and job satisfaction.

Thomas has been a nurse in your ICU for 10 years. In facilitating Thomas' professional development, the nurse manager would focus on: a. certification for the ICU environment. b. discussions about how Thomas can fit with role expectations and relationships. c. possible changes to other ICU environments. d. encouraging him to lead changes that leave long-term impacts after his retirement.

ANS: D Thomas is a mid-career professional. As such, you would expect him to be interested in honing areas of expertise (such as leadership or developing a deeper expertise in a particular area of ICU nursing) as opposed to gaining skills necessary for his work environment such as certification or becoming comfortable with his role and relationships in ICU, which would be critical to an early career nurse. Legacy building is characteristic of mid-career professionals.

A cover letter and a résumé together should be no longer than ________ page(s). a. two b. one c. four d. three

ANS: D Two pages are recommended for a résumé and one for a cover letter.

Complexity theories

Alter the traditional systems thinking approach by asserting that system behavior is unpredictable. This theory views change as emergent and highly influenced by all individuals and subsystems in an organization. Requires leaders to expand and respond to engaging dynamic change and focus on relationships rather than on prescribing and approaching change as a lock-step, pre-prescribed method.

12. Which management function is associated with advocacy? A) Awareness of current legislative efforts affecting nursing practice and organizational and unit management B) Role modeling proactive involvement in health-care policy through both formal and informal interactions with the media and legislative representatives C) Participation in professional nursing organizations and other groups that seek to advance the profession of nursing D) Assertively advocating on behalf of patients and subordinates when an intermediary is necessary

Ans: A Feedback: A management function associated with advocacy is awareness of current legislative efforts affecting nursing practice and organizational and unit management. Leadership roles include the following: role models' proactive involvement in health-care policy through both formal and informal interactions with the media and legislative representatives; participates in professional nursing organizations and other groups that seek to advance the profession of nursing; assertively advocate on behalf of patients and subordinates when an intermediary is necessary.

17. Whose rights do the legislative controls of nursing practice primarily protect? A) Patients B) Subordinates C) Managers D) Administrators

Ans: A Feedback: The legislative controls of nursing practice primarily protect the rights of patients. Protection of those identified by the other options while important is the primary focus of the nursing practice legislative bodies.

7. Which statement is true regarding political action committees (PACs)? A) They attempt to persuade legislators to vote in a particular way B) They are comprised of volunteers who are interested in politics C) Their focus is the passage of specific pieces of legislature D) They appear to have little influence with law makers

Ans: A Feedback: PACs of the Congress of Industrial Organizations attempt to persuade legislators to vote in a particular way. Lobbyists of the PAC may be members of a group interested in a particular law or paid agents of the group that wants a specific bill passed or defeated. Currently, PACs appear to have a significant amount of influence on law makers and the legislative process.

21. Which nursing intervention demonstrates attention to the nursing values central to patient advocacy? Select all that apply. A) Assuring the patient that their wishes will be respected regarding the care they receive B) Identifying the patient's religious dietary practices when discussing a prescribed diet C) Answering the patient's questions regarding the alternatives to a proposed procedure D) Suggesting to a patient that a social services consult would help with discharge needs

Ans: A, B, C, D Feedback: The nursing values central to advocacy emphasize caring, autonomy, respect, and empowerment. While appropriate, comforting a patient is not an act of advocacy in this situation.

16. Which intervention demonstrates that the nurse-manager is to advocate for patients in common areas of concern? Select all that apply. A) Staffing a unit with sufficient care providers B) Orienting staff to safely use a new patient lift C) Providing an in-service on culturally meeting end-of-life needs D) Arranging for patient consultations with members of the financial office

Ans: A, B, C, D Feedback: Common areas for managers to advocate for patients include distribution of resources; use of technology; end-of-life decisions; and health-care reimbursement. While aesthetics is an important consideration, it is not a common area for managers to act as advocators

15. Which are common areas requiring nurse-patient advocacy? Select all that apply. A) Securing patient consents B) Preventing medication errors C) Facilitating access to health-care services D) Respect for patient dignity and cultural values

Ans: A, B, C, D Feedback: Common areas requiring nurse-patient advocacy include inadequate patient consents; medical errors; access to health care; and respect for patient dignity and cultural values. While hospitalization is expensive, education on this subject is not considered a nursing advocacy role.

14. Which characteristic associated with being a patient increases the need for advocacy? Select all that apply. A) Loss of physical freedom as a result of extended hospitalizations B) Decreased independence due to physical or psychosocial limitations C) Physical or psychosocial limitations negatively impacted autonomy D) Illness tends to make an individual more vulnerable to unethical activities

Ans: A, B, C, D Feedback: Patient advocacy is necessary because disease almost always results in decreased independence, loss of freedom, and interference with the ability to make choices autonomously. In addition, aging, as well as physical, mental, or social disability may make individuals more vulnerable and in need of advocacy. It is not true that cognitive function is impaired as a result of chronic and/or acute illness.

9. Which strategies would demonstrate an understanding of effective preparation for a television interview to discuss issues affecting nursing today? Select all that apply A) Be focused on key points. B) Introduce statistics that support the key points C) Repeat key points several times during the interview D) Provide background material related to the key points

Ans: A, B, C, D Feedback: The best strategies for a television interview are to be prepared and to stick to three or four key points that will drive home your message, and repeat them during the interview. Concentrating on popular points is not necessarily where the focus should be directed.

23. Which statement correctly identifies the composition of a letter to a legislator? Select all that apply. A) Identify yourself as a constituent in the first paragraph B) State your reason for writing in the second paragraph C) Sign the letter and include your contact information D) Address the legislator by title

Ans: A, C, D Feedback: Personal letters are more influential than formal letters, and the tone should be formal but polite. The letter should also be concise (not more than one page). Be sure to address the legislator properly by title. Establish your credibility early in the letter as both a constituent and a health-care expert. State your reason for writing the letter in the first paragraph, and refer to the specific bill that you are writing about. Then, state your position on the issue and give personal examples as necessary to support your position. Offer your assistance as a resource person for additional information. Sign the letter, including your name and contact information. Remember to be persistent, and write legislators repeatedly who are undecided on an issue. Display 6.5 displays a format common to letters written to legislators.

8. Which factor has the greatest influence on a legislator? A) Form letters from group members B) Individual phone calls from constituents C) Meeting with a strong collective political group D) A personal visit by an individual constituent

Ans: C Feedback: Legislators and policy makers generally are more willing to deal with a group rather than individuals; thus, joining and supporting professional organizations allow nurses to become active in lobbying for a stronger nurse practice act or for the creation or expansion of advanced nursing roles.

1. Which statement regarding the responsibility of a manager in advocacy is accurate? A) Advocacy is a management function and not a leadership role B) Managers advocate only as needed to meet organizational goals C) Managers should advocate for patients as well as subordinates D) Professional advocacy is not a primary concern for most managers

Ans: C Feedback: Advocacy is helping others to grow and self-actualize. The manager must be an advocate for patients, subordinates, and the nursing profession. The remaining statements are not accurate descriptions of the manager's role as advocate.

18. Patients were often denied basic human rights until when? A) 1940s B) 1950s C) 1960s D) 1970s

Ans: C Feedback: Until the 1960s patients had few rights; in fact, patients before then often were denied basic human rights. Conditions improved by the 1970s.

19. The Patient's Bill of Rights established by which body has the most legal authority? A) A health-care organization B) A professional medical organization C) A regulation professional nursing organization D) A specific state

Ans: D Feedback: A bill of rights that has become law or state regulation has the most legal authority because it provides the patient with legal recourse. A bill of rights issued by health-care organization and professional associations is not legally binding but may influence federal or state funding and certainly should be considered professionally binding.

6. Which situation is a reality for whistle-blowers? A) Retaliation against them is illegal B) Fellow workers are supportive of them C) Federal and State law protects them D) They are often afraid to speak out

Ans: D Feedback: Although whistle-blower protection has been advocated for at the federal level and has been passed in some states, many employees are reluctant to report unsafe conditions for fear of retaliation. Nurses should check with their state association to assess the status of whistle-blower protection in their state. Retaliation and resentment still exist against these individuals in many situations.

What statement is true concerning nurses as a political body? A) They are very committed to both their profession and the political process B) Many are active members of the American Nurses Association (ANA) C) Their primary focus is to speak out on consumer health-related issues D) They have not yet recognized the full potential of political activity

Ans: D Feedback: As a whole, the nursing profession has not yet recognized the full potential of collective political activity. Nurses must exert their collective influence and make their concerns known to policy makers before they can have a major impact on political and legislative outcomes. The remaining options are not true of nurses in general.

3. Even after a lengthy discussion concerning the ramifications, a patient wants to leave the hospital against medical advice (AMA). What would the nurse do in the role of patient advocate? A) Encourage the patient to wait until morning to leave the hospital B) Ask family members to help talk the patient out of the decision C) Call the security guard to escort the patient off hospital property D) Make sure the patient has appropriate follow-up appointments

Ans: D Feedback: As advocate for this patient, you have made sure he is informed of the ramifications of leaving AMA; however, it is a patient's right to refuse treatment. The other options fail to recognize the patient's right to make autonomous choices.

The nurse educator of the pediatric unit determines that vital signs are frequently not being documented when children return from surgery. According to quality improvement (QI), to correct the problem, the educator, in consultation with the patient care manager, would initially do which of the following? a. Talk to the staff individually to determine why this is occurring. b. Call a meeting of all staff to discuss this issue. c. Have a group of staff nurses review the established standards of care for postoperative patients. d. Document which staff members are not recording vital signs, and write them up

Call a meeting of all staff to discuss this issue

A nurse is participating in a baccalaureate course. For the class, she has to attend the legislative session regarding the new role of medication assistants. Nurses should be involved in shaping public policy primarily because: a. Involvement will enable nurses to take over the healthcare system at some point in the future. b. Other healthcare professions are less concerned about the essential needs of clients. c. Such activities are important career builders for nurses who seek top-level executive positions. d. They are closest to the front line of health care and see how it affects clients and families.

D

Which of the following interactions is MOST consistent with the idea of networking? a. Meet with the same colleagues daily to have coffee and share concerns about the workplace and stories about colleagues. b. Join an online workplace forum to gain ideas about how to handle workplace conflict. c. Suggest that you and a new team member meet after work for coffee to review unit guidelines. d. Join a nurse executive informal lunch meeting to meet other executives for support and for sharing ideas of expertise.

D

Which of the following is the best example of skilled negotiation? a. Linda, the manager on pediatrics, takes a proposal to her supervisor, outlining the benefits of a walk-in preoperative area for children. b. Kim, RN, asks for leave to pursue a semester of full-time study in her graduate program. She proposes to accept less popular rotations during peak vacation time, in return. c. George, the head nurse in ER, asks for additional staff for his department and points out the benefits of being able to keep patients longer. d. Jerry speaks with his supervisor about his supervisor's concerns related to bedside reporting before presenting a proposal to change this process.

D

A healthcare organization is committed to improving patient outcomes as part of the quality improvement (QI) process and examines its executive structure and organizational design. This approach recognizes which model of QI? a.Donabedian b.Benchmarking c.Employee involvement and innovation d. QSEN

Donabedian

Hospital ABCD is a Magnet™ hospital. This designation has been applied to Hospital ABCD because it: a. Facilitates active staff participation in decision making related to quality nursing care. b. Has implemented a graduate nurse orientation program. c. Espouses commitment to excellence in patient care. d. Is establishing career ladders for nurses.

Facilitates active staff participation in decision making related to quality nursing care

Through the QI process, the need to transform and change the admissions process across administrative and patient care units is identified. In this particular situation, what method of data organization will be most effective? a. Flowchart b.Histogram c.Narrative d.Line graphs

Flowchart

At Hospital Ajax, staff members are reluctant to admit to medication errors because of previous litigation and a culture that seeks to assign blame. This culture demonstrates: a.QM principles that emphasize customer safety. b.A deep concern with improvement of quality and processes. c.Effective employee orientation and development in relation to QM. d.Goals that are inconsistent with QM

Goals that are inconsistent with QM

With the rise in workplace violence in the emergency department, the nurse manager decides that she should work with the risk manager in violence prevention. The nurse manager should: a.Request all staff to accept new risk management practices. b.Hold staff accountable for safe practices. c.Document inappropriate behavior. d.Hire more police security

Hold staff accountable for safe practices

The nurse manager is concerned about the negative ratings her unit has received on patient satisfaction surveys. The first step in addressing this issue from the point of view of quality improvement is to: a.Assemble a team. b.Identify a clinical activity for review. c.Establish a benchmark. d.Establish outcomes

Identify a clinical activity for review

A new graduate is asked to serve on the hospital's quality improvement (QI) committee. The nurse understands that the first step in quality improvement is to: a.Collect data to determine whether standards are being met. b.Implement a plan to correct the problem. c.Identify the standard. d.Determine whether the findings warrant correction.

Identify the standard

A nurse is explaining the pediatric unit's quality improvement (QI) program to a newly employed nurse. Which of the following would the nurse include as the primary purpose of QI programs? a. Evaluation of staff members' performances b. Determination of the appropriateness of standards c. Improvement in patient outcomes d. Preparation for accreditation of the organization by the Joint Commission on Accreditation of Healthcare Organizations (JCAHO)

Improvement in patient outcomes

The chief executive officer asks the nurse manager of the telemetry unit to justify the disproportionately high number of registered nurses on the telemetry unit. The nurse manager explains that nursing research has validated which statement about a low nurse-to-patient ratio? It: a.Promotes teamwork among healthcare providers. b.Increases adverse events. c.Improves outcomes. d.Contributes to duplication of services

Improves outcomes

Change agents

Individuals with formal or informal legitimate power whose purpose is to initiate, champion, and direct or guide change.

At Hospital Ajax, there has been a 20% increase in instruments and sponges being left in patients during surgery and surgeries on the wrong limbs. These are known as: a.Sentinel events. b.Medically sensitive events. c.Nurse-sensitive events. d.Never events.

Never-events

A nurse manager wants to decrease the number of medication errors that occur in her department. The manager arranges a meeting with the staff to discuss the issue. The manager conveys a total quality management philosophy by: a.Explaining to the staff that disciplinary action will be taken in cases of additional errors. b.Recommending that a multidisciplinary team should assess the root cause of errors in medication. c.Suggesting that the pharmacy department should explore its role in the problem. d.Changing the unit policy to allow a certain number of medication errors per year without penalty.

Recommending that a multidisciplinary team should assess the root cause of errors in medication

The nurse gives an inaccurate dose of medication to a patient. After assessment of the patient, the nurse completes an incident report. The nurse notifies the nursing supervisor of the medication error and calls the physician to report the occurrence. The nurse who administered the inaccurate medication understands that: a. The error will result in suspension. b. An incident report is optional for an event that does not result in injury. c. The error will be documented in her personnel file. d. Risk management programs are not designed to assign blame.

Risk management programs are not designed to assign blame

The unit is shifting from primary nursing to a team model in an effort to contain costs. Staff members are angry and ask for a meeting to discuss the change. After hearing their concerns related to reduction in professional autonomy and care quality, you: a. Acknowledge the loss. b. Explain the reasons for change, emphasizing the need to reduce costs. c. Repeat the information several times, giving detailed budget overviews. d. Adjourn the meeting and provide explanation through e-mail.

a. Acknowledge the loss. Visioning involves engaging with others to assess the current reality, specify the end point, and then strategize to reduce differences. This requires trusting relationships that acknowledge the differences in values and ideas. When done well, the nurse and the nurses within a unit experience creative tension that inspires working in concert to achieve desired goals.

Which of the following would managers and staff review annually in order to ensure compliance with the Joint Commission (TJC) to improve patient safety? a. Appropriateness of charting terms and abbreviations b. Nursing hours per patient c. Acuity of patient admissions d. Wait times for care

a. Appropriateness of charting terms and abbreviations The Joint Commission issues setting-specific patient goals annually, as well as a list of "do-not-use" terms, abbreviations, and symbols and sentinel events.

At a second negotiation session, the unit manager and staff nurse are unable to reach a resolution. It would now be best to: a. Arrange another meeting in a week's time so as to allow a cooling-off period. b. Turn the dispute over to the director of nursing. c. Insist that participants continue to talk until a resolution has been reached. d. Back the unit manager's actions and end the dispute.

a. Arrange another meeting in a week's time so as to allow a cooling-off period. According to the principles outlined by Ury, Brett, and Goldberg, a "cooling-off" period is recommended if resolution fails.

Which of the following patients would be at greatest risk in a healthcare visit (select all that apply)? a. Clyde requires an anticoagulant. He tells the nurse about his medications. He does not include an herbal supplement. b. George is very shy and withdrawn. He asks the nurse to leave him alone. c. Sarah is a new parent who finds that nurses on the children's unit are very helpful. She is eager to accept all suggestions, including those that she does not yet understand. d. Claude is booked for bowel surgery. His doctor explains about the colostomy. Later, Claude tells his wife that he really doesn't know what the doctor meant by colostomy.

a. Clyde requires an anticoagulant. He tells the nurse about his medications. He does not include an herbal supplement. c. Sarah is a new parent who finds that nurses on the children's unit are very helpful. She is eager to accept all suggestions, including those that she does not yet understand. d. Claude is booked for bowel surgery. His doctor explains about the colostomy. Later, Claude tells his wife that he really doesn't know what the doctor meant by colostomy. Safer health care involves the patient as an active consumer who keeps and brings a list of all medications, including natural remedies, and questions if there are doubts, concerns, or lack of understanding.

You notice that Sally, a student on your unit, is giving information to an anxious young teen who seems very uncertain about preparation for an upper GI series. After Sally leaves the room, you ask her how she thought her conversation with the patient went and: a. Encourage her to ask the patient if he has questions or concerns about the procedure. b. Advise her to consider providing the patient with more information. c. Suggest that she leave some brochures on the procedure with the patient. d. Suggest that she also provide teaching to the adolescent's parents.

a. Encourage her to ask the patient if he has questions or concerns about the procedure. The Five Steps to Safer Health Care for Patients includes the step of asking questions if there are doubts or concerns. The nurse can encourage patients to take a larger role in care by taking these steps and by providing patients with coaching in the steps.

The manager of a surgical area has a vision for the future that requires the addition of RN assistants or unlicensed persons to feed, bathe, and walk patients. The RNs on the staff have always practiced in a primary nursing-delivery system and are very resistant to this idea. The best initial strategy in this situation would include: a. Exploring the values of the RN group in relationship to this change. b. Leaving the RNs alone for a time so they can think about the change before they are approached again. c. Dropping the idea and trying for the change in another year or so. d. Hiring the assistants and allowing the RNs to see what good additions they are.

a. Exploring the values of the RN group in relationship to this change. Influencing others requires emotional intelligence in domains such as empathy, handling relationships, deepening self-awareness in self and others, motivating others, and managing emotions. Motivating others recognizes that values are powerful forces that influence acceptance of change. Leaving the RNs alone for a period of time before implementation does not provide opportunity to explore different perspectives and values. Avoiding discussion until the team changes may not promote adoption of the change until there is opportunity to explore perspectives and values related to the change. Hiring of the assistants demonstrates lack of empathy for the perspectives of the RN staff.

As a manager in a new nursing home, where might you consult for guidance and evidence to support the development of safe patient practices? a. Hospitals b. Business c. Industry d. Outpatient clinics

a. Hospitals Practices that were once mostly studied in hospital settings are now scrutinized for implementation in other settings, such as outpatient clinics, rural settings, and nursing homes.

You pull staff together to assess a situation in which the family of a seriously ill patient is anxious and is absorbing a great deal of staff time in consultation, discussion, and questioning of treatment decisions. Staff members are becoming distanced from the family. After inviting the concerns of staff, you explain that the organization values patient-centered care and suggest that evidence supports that acting as an advocate and a listener is helpful to families. You ask the staff for ideas as to strategies that are effectively patient-centered in these situations. In this situation, you are taking on which role? a. Leadership b. Management c. Follower d. Visionary

a. Leadership As a leader, you provide and communicate vision and direction based on evidence and experience, and you engage others in decision making that moves them toward the vision with a reasonable level of risk taking.

A family is keeping vigil at a critically ill patient's bedside. Other, distant family members, not yet able to come, call the unit continuously, asking for updates and wanting to express concern. You speak with the distant family members and suggest that you are going to refer them to the hospital social worker, whose role is to work with such situations. What role are you assuming through this action? a. Manager b. Leader c. Follower d. Laissez faire

a. Manager As a manager, you are concerned with managing and coordinating resources to achieve outcomes in accordance with established clinical processes. Referral to a social worker alleviates demand on staff time and is consistent with hospital procedures.

If you are supporting the steps in the AHRQ document "Five Steps to Safer Health Care," you would ensure that: a. Patients are actively encouraged to make decisions related to care. b. Rules and decisions are made through centralized processes. c. You monitor the performance of each staff member closely. d. Preference is given to increasing staff numbers rather than staff credentials.

a. Patients are actively encouraged to make decisions related to care The Agency for Healthcare Research and Quality (AHRQ) outlines "Five Steps to Safer Health Care," which suggests that safe, patient-centered care is facilitated by assisting patients to become active partners in their own care.

Sarah is a second-year nursing student. The clinical instructor overhears Sarah telling a patient that she "always" checks patients' bracelets before giving medication and she is not sure how the nurses on the unit "get away with" not making more errors than they do. The clinical instructor pulls Sarah aside and explores with her how her communication might affect the patient and what it reflects about her beliefs related to the team. The actions of the clinical instructor reflect competencies outlined by: a. QSEN. b. IHI. c. DNV/NIAHO. d. AHRQ.

a. QSEN. The Quality and Safety Education for Nurses (QSEN) project provides resources related to competencies that prelicensure and graduate students need to develop to serve as safe practitioners. These competencies include leading and managing, teamwork and collaboration.

A charge nurse on a busy 40-bed medical/surgical unit is approached by a family member who begins to complain loudly about the quality of care his mother is receiving. His behavior is so disruptive that it is overheard by staff, physicians, and other visitors. The family member rejects any attempt to intervene therapeutically to resolve the issue. He leaves the unit abruptly, and the nurse is left feeling frustrated. Which behavior by the charge nurse best illustrates refined leadership skills in an emotionally intelligent practitioner? a. Reflect to gain insight into how the situation could be handled differently in the future. b. Try to catch up with the angry family member to resolve the concern. c. Discuss the concern with the patient after the family member has left. d. Notify nursing administration of the situation.

a. Reflect to gain insight into how the situation could be handled differently in the future. Goleman suggests that emotional intelligence involves insight and being able to step outside of the situation to envision the context of what is happening as well as being able to manage emotions such as frustration effectively

As a manager, the development of your decision-making skills related to safe patient care is facilitated by: a. Regular reflection on decisions. b. A culture of perfectionism. c. Recognition of who should be held responsible for individual errors. d. A culture of trust between the staff and you.

a. Regular reflection on decisions. Reflection on how well decisions were enacted enables knowledge of the complexity of situations and ramifications of the decisions made. Reflection enables elimination of strategies and methods that are inappropriate in meeting needs and aids in narrowing choices of best actions to take.

In accordance with changes by the Joint Commission (TJC), Pleasant Valley Hospital amends its safety practices and policies to emphasize: a. Safety goals specific to Pleasant Valley. b. Decision-making processes. c. Sufficient staffing for safe care. d. Increased numbers of baccalaureate-prepared RNs.

a. Safety goals specific to Pleasant Valley. When the TJC, a not-for-profit organization that accredits healthcare organizations, changed its focus from processes to outcomes, it emphasized patient safety and issues setting-specific annual patient safety goals.

A client requires an appendectomy. The surgeon explains the procedure and asks the client to sign the consent. The patient speaks very little English and looks worried. As a nurse, you would: a. Suggest that an interpreter explain the procedure to the client and answer any questions. b. Ask the client if he has any questions. c. Draw a picture to show the incision. d. Not intervene.

a. Suggest that an interpreter explain the procedure to the client and answer any questions. The Five Steps to Safer Patient Care identifies that encouraging patients to ask questions when there are doubts and concerns and ensuring understanding before surgery is performed are ways in which nurses can support patients in having greater influence in their own care. In this situation, asking an interpreter to help enables access to information for the patient and active assessment of his understanding.

As a patient care advocate, you regularly coach patients on how to stay safe in health care by educating them about: a. The need to understand and record all medications being taken. b. Bringing their own linens and other personal items to the hospital. c. Washing hands frequently while in a healthcare environment and using a hand sanitizer. d. Following closely the directions and orders of healthcare providers.

a. The need to understand and record all medications being taken. The Five Steps to Safer Health Care for patients include keeping a list of medications that patients are taking.

During a staff meeting held to discuss developing a mission statement for the unit, the idea of placing patient needs first is: a. Empowering. b. A leadership tag. c. A symbol. d. A management task.

b. A leadership tag. According to complexity theory, leadership tags, which are similar to values, reflect the patient-centered philosophies and values-driven characteristics that define an organization and give it personality.

As a nurse, you are responsible for teaching ostomy patients self-management skills postoperatively. Mr. Jones is 2 days postoperative after an abdominal perineal resection. In spite of patient-controlled analgesia, Mr. Jones acknowledges inadequate pain relief and rates his pain as an 8, utilizing a 0-to-10 pain scale. When you approach him for teaching, he turns away and closes his eyes. Which approach incorporating Maslow's hierarchy of needs motivational theory is most appropriate in this situation? a. Tell him you will let him rest today and see if he is feeling better tomorrow. b. Intervene to improve his pain management control and return later in the day to reassess his readiness to learn. c. Talk to the social worker about arranging home healthcare services at discharge to assist with teaching. d. Gently approach Mr. Jones and inform him of the importance of participating in postoperative teaching today because he may be discharged within several days.

b. Intervene to improve his pain management control and return later in the day to reassess his readiness to learn. Unless Mr. Jones' basic need for comfort is met, he will not be motivated toward higher-order needs related to self-esteem and competence.

Chart audits have revealed significant omissions of data that could have legal and funding guidelines. As the unit manager, you meet with the staff to discuss audit findings and to find approaches that will address the gaps in charting and achieve desired goals. This is an example of: a. Leadership. b. Management. c. Decision making. d. Vision.

b. Management. The process of guiding others to meet established goals, outcomes, and procedures is management. This can require collaborative decision making to determine how best to reach pre-determined goals and follow established practices.

You overhear a newly graduated RN telling one of your colleagues that leadership and management belong to the unit manager and not to her. As a nursing colleague, your response demonstrates understanding that the perception of the new graduate: a. Is correct. Leadership is not the role of the staff nurse. b. Would benefit from further understanding of her role as a professional, whose influence may affect the decision making of patients, colleagues, and other professionals. c. Has been influenced by nurse leaders and managers who leave for other positions. d. Is related to the general perception that nurse leaders and managers are not satisfied in their jobs.

b. Would benefit from further understanding of her role as a professional, whose influence may affect the decision making of patients, colleagues, and other professionals. Care coordination that involves the intersection of individual, family, and community-based needs requires that nurses have self-confidence, knowledge of organizations and health systems, and an inner desire to lead and manage. There is often a view that leadership is isolated to those holding managerial positions and that a direct care nurse is subject to following by adhering to the direction of others. Such views fail to acknowledge that to be a nurse requires each licensed individual to lead, manage, and follow when practicing at the point-of-care and beyond.

You recently acquired a position as a unit manager. During your time on the unit, you have formed a strong social network among your staff, have promoted the development of relationships between your staff and workers in other areas of the organization, and have formed relationships that generate ideas from patient organizations and the local nursing education program. According to complexity theory, you are engaging which principle? a. Empowerment b. Systematic thinking c. Development of networks d. Bottom-up interactions

c. Development of networks According to complexity theory, social networks evolve around areas of common interest and are able to respond to problems in creative and novel ways.

The Rehabilitation Unit at Pleasant Valley Hospital has a high number of falls. Which of the following interventions might assist to reduce the number of falls on the unit? a. Determining who is responsible for the falls b. Strengthening unit policies to avoid inappropriate admissions c. Encouraging involvement of nurses in education related to falls and safety d. Ensuring that patients are appropriately restrained if they are at risk for falls

c. Encouraging involvement of nurses in education related to falls and safety The IOM (2010) emphasizes the need for nurses to engage in lifelong learning and to use evidence and best practices to inform practice and ensure safety.

To increase safety in patient care areas of the Valley Hospital, the executive begins by: a. Asking the community what the safety issues are. b. Consulting with a management expert about staffing schedules. c. Ensuring that the senior nursing officer attends the board meetings. d. Instituting improved practices to reduce needle-stick injuries.

c. Ensuring that the senior nursing officer attends the board meetings The IOM report (2004) highlighted the importance of the attendance of the senior nurse executive at board meetings to be a key spokesperson on safety and quality issues.

In response to the situation in Question 15, you approach the unit manager to apprise her of your concerns that the family dynamics of the patient involved may lead to staff-family and patient-family conflicts. You suggest that the physician may need to discuss the treatment plan with the family. The unit manager advises that he will arrange this discussion. If, after the meeting with family members, this is identified as a desired approach, you support the manager's decision. Your actions indicate that you are acting in what role? a. Leadership b. Management c. Follower d. Evidence-based

c. Follower In the followership role, you bring to the manager your concerns about potential litigation and maximization of outcomes and accept the direction given by the manager in response to your concerns.

Mary, an 85-year-old patient with cognitive impairment and gross instability, wanders continuously. Lately, she has fallen twice, and the family demands that she be restrained. As the unit manager, you have initiated a least restraint practice. An appropriate action in this situation would be: a. Setting up a nursing team meeting to review practices. b. Calling the family to inform them of the practice. c. Initiating a multidisciplinary and family meeting to focus on Mary's needs. d. Restraining Mary to satisfy the family's wishes.

c. Initiating a multidisciplinary and family meeting to focus on Mary's needs. Crossing the Quality Chasm emphasizes the importance of rendering care with the client (client-centered) rather than to the client. In this situation, the patient includes family in transparent discussions about quality needs and takes a team approach that involves healthcare professionals, the family, Mary's needs, and evidence associated with safe practice.

You walk into Mr. Smith's room and find him yelling at the LPN, Miss Jones. He is obviously very upset and after you speak with him regarding his behavior, you determine that he has not slept for three nights because of unrelieved pain levels. The LPN is very upset with Mr. Smith and calls him an "ugly, old man." You acknowledge her feelings and concerns and then suggest that Mr. Smith's behavior was aggressive, but is related to lack of sleep and to pain. "Can you both, together with Mr. Smith, determine triggers for the pain and effective approaches to controlling his pain?" This approach demonstrates: a. Lack of empathy and understanding for Miss Jones. b. Concern with placating Mr. Smith. c. Leadership behavior. d. Management behavior.

c. Leadership behavior. The situation between Mr. Smith and Miss Jones is a complex situation involving unrelieved patient symptoms and aggressiveness toward a staff member. Providing engaged, collaborative guidance and decision making in a complex situation where there is no standardized solution reflects leadership.

As the manager, you have been asked to implement an evidence-based approach to teach ostomy patients self-management skills postoperatively that is being operationalized throughout your organization. Which of the following illustrates effective leadership? a. The training modules are left in the staff room for times when staff are available. b. The current approach is continued because it is also evidence-based and is more familiar to staff. c. You decide to implement the approach at a later date because of feedback from the RNs that the new approach takes too much time.

d. A RN who is already familiar with the new approach volunteers to take the lead in mentoring and teaching others how to implement it. Followership occurs when there is acquiescence to a peer who is leading in a setting where a team has gathered to ensure the best clinical decision making and actions are taken to achieve clinical or organizational outcomes. Followership promotes good clinical decisions and use of clinical resources.

After consulting with practice environments about quality and safety concerns in health care, the Dean of Health Programs at U.S. University develops: a. A nursing program that emphasizes the development of a strong disciplinary identity. b. Programming that stresses discipline-based research. c. Partnerships with health care to develop software for the reporting of adverse events. d. An interdisciplinary program for nurses, pharmacists, and medical practitioners that emphasizes collaborative learning teams.

d. An interdisciplinary program for nurses, pharmacists, and medical practitioners that emphasizes collaborative learning teams. Health Professions Education identified that education related to health disciplines in silos leads to compromised communication and inability to function as an integrated whole for patient-centered care.

In an effort to control costs and maximize revenues, the Rehabilitation Unit at Cross Hospital reduced the number of its managers and increased the number of units for which each manager was responsible. Within a year, the number of adverse events on the units had doubled. This may be attributable to: a. The overload of staff nurses. b. Resistance to change by staff. c. A change in reporting systems. d. Fewer clinical leaders to remove barriers to care.

d. Fewer clinical leaders to remove barriers to care. Eliminating barriers to the implementation of best practices is the role of managers and leaders. When there are insufficient resources for leadership to encourage a culture in which evidence-based practice is embraced, frontline nurses recognize this as a stumbling block for delivering quality care.

10. As a nurse manager on the West Surgery Unit, you are interested in increasing patient safety and reducing morbidity and mortality on your unit. Which of the following recommendations would be consistent with the IOM The Future of Nursing report? a. Careful screening of nursing staff for substance use and abuse b. Increased RN staffing on the unit c. Salary and benefits that reflect nursing accountabilities d. Increase in the percentage of baccalaureate-prepared nurses to 80%

d. Increase in the percentage of baccalaureate-prepared nurses to 80% The Future of Nursing advocates for having 80% of the nursing population at a baccalaureate-prepared level. This recommendation reflects research that suggests that improved mortality and morbidity rates occur with a better educated workforce.

As the manager on an acute care medical unit, you note that the incidence of medication errors has increased since the implementation of staffing changes. As part of your strategy to reduce errors, it is important to a. Re-visit reporting standards for medication errors in your organization. b. Ensure that medication errors are consistently reported. c. Provide staff with additional education related to safe practice in medication administration. d. Involve RN staff in determining reasons for errors and practice solutions to increase the safety of medication administration.

d. Involve RN staff in determining reasons for errors and practice solutions to increase the safety of medication administration. Keeping Patients Safe: Transforming the Work Environment of Nurses (2004) identified many past practices that had a negative impact on nurses, and thus on patients, and recommended the inclusion of nurses in direct care in decision making involving their practice. Future of Nursing: Leading Change, Advancing Health (2010) also emphasizes the role of nurses as leaders in changes that improve health.

According to the complexity theory, which of the following should be the focus of measurement? a. Cost per hospital day b. Bed utilization c. Infection rates d. Staff morale and budgets

d. Staff morale and budgets According to complexity theory and the principle "Think systematically," you cannot ignore objective data or nonmeasurable data, as both inform decisions.

After assessing an older adult patient in long-term care who has been slowly deteriorating for weeks, the nurse manager calls the family and asks them to come in, as the patient is dying. The nurse manager's decision and actions are based on: a. An established clinical pathway. b. Confirmatory scientific evidence. c. Unit protocol. d. Tacit knowledge / experience.

d. Tacit knowledge / experience. The nurse manager is employing knowledge and experience in determining that the patient is dying, because the course of dying is not standardized and cannot be determined by clinical pathways.

On the West Surgery unit, you want to institute a new system for checking armbands that evidence suggests may increase safety in medication administration. The system involves technology. Which of the following strategies may assist with rapid adoption of the technology and system? a. Employ a centralized decision-making approach. b. Use simulators for initial practice to build confidence. c. Bring in a nurse consultant who is familiar with the technology. d. Use early adopters among the staff as leaders and role models in implementation.

d. Use early adopters among the staff as leaders and role models in implementation. The Institute for Healthcare Improvement (IHI) is dedicated to rapid improvement in patient care through a variety of mechanisms such as rapid cycle change. Rapid cycle change diffuses innovation and changes quickly through early adopters who share information and energy over time and act as role models for others.

A unit manager of a 25-bed medical/surgical area receives a phone call from a nurse who has called in sick five times in the past month. He tells the manager that he very much wants to come to work when scheduled but must often care for his wife, who is undergoing treatment for breast cancer. According to Maslow's need hierarchy theory, what would be the best approach to satisfying the needs of this nurse, other staff, and patients? a. Line up agency nurses who can be called in to work on short notice. b. Place the nurse on unpaid leave for the remainder of his wife's treatment. c. Sympathize with the nurse's dilemma and let the charge nurse know that this nurse may be calling in frequently in the future. d. Work with the nurse, staffing office, and other nurses to arrange his scheduled days off around his wife's treatments.

d. Work with the nurse, staffing office, and other nurses to arrange his scheduled days off around his wife's treatments. Placing the nurse on unpaid leave may threaten the nurse's capacity to meet physiologic needs and demotivate the nurse. Unsatisfactory coverage of shifts on short notice could affect patient care and threaten the needs of staff to feel competent. Arranging the schedule around the wife's needs meets the needs of the staff and of patients while satisfying the nurse's need for affiliation.

5. The IOM Health Professions Education report highlighted patient safety concerns as: a. A normal risk in professional practice. b. A result of disciplinary silos. c. A reflection of frontline staff. d. Related to systems errors.

b. A result of disciplinary silos. The IOM Health Professions Education report (2004) highlighted the education of health disciplines in silos as a major concern in patient safety and endorsed five recommendations.

As the head of a nursing program, you consistently invite the ideas of your team about innovations in teaching, community partnerships, and curriculum design and invite participation in decisions. Many of these ideas have been implemented successfully, and your staff members are keen to try on other ideas. You are employing _____ leadership. a. Situational b. Trait-based c. Contingency-based d. Transformational

d. Transformational Transformational leadership involves attending to the needs and motives of followers, which results in creativity, improvement, and employee development.

A unit manager watches a new RN graduate interacting with a patient. When the RN comes out of the room, the unit manager says, "I don't know what they taught you in your nursing program, but if I see you do that again, I will write you up." This example demonstrates: a. Coercive use of power. b. Appropriate application of control. c. Use of informatory power. d. Use of power to provide coaching.

A

24. Leslie, a UNP, transfers a patient while using improper technique. The patient is injured, and as a result, a suit is launched in which both Sarah (the delegator) and Leslie (the delegatee) are named. Sarah is named in the suit because she: a. retains accountability for the care of the patient. b. worked the same shift as Leslie. c. has passive accountability for delegation. d. retains accountability for the outcomes of care for the patient.

ANS: D Whenever care is provided by someone other than a registered nurse, accountability for care remains with the manager/delegator even though others provide aspects of care.

Sally (RN) and Melissa (RN) have shared an ongoing conflict since the first day that Melissa worked on the unit. Sally has confided to another colleague that she doesn't even know why the conflict started or what it was about. This is an example of: a. how expectations and objectives need to be made clear in team situations. b. the need to encourage open discussion of disagreements in opinions. c. the importance of involving all staff in discussions in group settings. d. the enduring nature of first impressions.

ANS: D First impressions are lasting and, as Sally indicates to her colleagues, are often an unconscious response.

The number of adverse events such as falls and pressure ulcers on the unit is increasing. An ideal staffing plan to address this issue would include which of the following? Increasing the: a. total number of staff on the unit. b. staff and RN hours per patient. c. total number of staff and implementing 12-hour shifts. d. number of RNs and number of RNs with experience on the unit.

ANS: D A number of studies have identified that adverse events such as falls and pressure ulcers can be reduced by increasing the number of RNs on a unit (relative to other personnel) and utilizing experienced RNs. Overtime and 12-hour shifts are linked to greater incidence of errors

The American Nurses Association has advocated for the Registered Nurse Safe Staffing Act. This legislation is based on ANA Safe Staffing Principles and takes what factors into consideration for planning staffing on a nursing unit? a. Patient acuity and complexity b. Education and training of the nurses c. Technology available and use on the unit d. All factors listed are to be considered

ANS: D ANA has opted to support the nurse staffing committee as the approach to ensure safe staffing. For the last decade, the ANA has advocated for a Registered Nurse Safe Staffing Act. The legislation is based upon the ANA Safe Staffing Principles (Box 13-2) and considers the following: RN educational preparation, professional certification, and level of clinical experience The number and capacity of available healthcare personnel Geography of the unit Available technology Intensity, complexity, and stability of patients

Before beginning her own nursing agency, a nurse worked with other temporary nursing agencies in nine states and multiple agencies. She had set a personal goal to start her own agency. What type of motivation does this exemplify in career success? a. Linear b. Spiral c. External d. Internal

ANS: D This is an example of internally motivated career success for the nurse. She set a goal and achieved the goal, meeting her own internal focus and resulting in her career success.

18. John is a circulating nurse in the operating room. He is usually assigned to general surgery, but on this day he is assigned to the orthopedic room. He is unfamiliar with the routines and studies the doctor's preference cards before each patient. The fourth patient comes into the room and John prepares a site for a biopsy using a Betadine solution. The surgeon prefers another solution. He notices what John has done and immediately corrects him by rudely insulting John. What is the appropriate approach to conflict resolution in this example? a. Collaboration b. Compromising c. Avoiding d. Withdraw

ANS: B Compromise involves negotiation or an exchange of concessions and supports a balance of power.

9. The day shift nurse asks an LPN/LVN to complete a task for a patient. The day shift nurse is engaging in what function? a. Delegating b. Assigning c. Sharing d. Authorizing

ANS: B Delegation refers to transfer of responsibility for work; the day shift nurse retains accountability for the outcomes of patient care therefore is using assigning of the task rather than delegation.

The parents of a toddler who dies after being brought to the ER launch a lawsuit, claiming that the failure of nurses to pursue concerns related to their son's deteriorating condition contributed to his death. The senior nurse executive is named in the suit: a. As a global respondent. b. Under the doctrine of respondeat superior. c. As a frivolous action. d. Under the element of causation.

ANS: B Known as vicarious liability, the doctrine of respondeat superior makes employers accountable for the negligence of their employees, using the rationale that the employee could not have been in a position to have caused the wrongdoing unless hired by the employer.

By following a shared leadership model, the nurse manager believes that staff members will learn to function synergistically. Some teams function synergistically because members: a. do not volunteer unwanted information. b. actively listen to each other. c. listen to the person who believes he or she is an expert. d. do not speak unless they are absolutely sure they are correct in their views.

ANS: B Active listening in a group creates synergy in that team members really hear one another's ideas and share in decision making.

A nurse is applying for a new position. This position is one in which she will serve as a liaison between a hospital and a school of nursing. The nurse must update her résumé to include her teaching experience. The goal of creating a curriculum vitae is to: a. have a listing of facts about your professional life. b. provide potential employers with information about where you are in your career. c. respond quickly whenever a position becomes available. d. be certain you can recall facts for a prospective position.

ANS: B A curriculum vitae provides an all-inclusive but not detailed listing of your professional life. This approach allows a prospective employer to find the most recent information quickly and gain a sense of where you are in your career.

To enhance team leadership skills for your team leaders, you arrange opportunities for: a. certification. b. continuing education. c. graduate courses. d. volunteerism.

ANS: B Continuing education provides systematic learning opportunities that augment existing skills and knowledge for delivery of quality care and advancement of career goals. Graduate and certification courses provide advanced knowledge and skills.

9. Jane has transferred from the ICU to the CCU. She is very set in the way she makes assignments and encourages her new peers to adopt this method without sharing the rationale for why it is better. This is a good example of a process and procedure that creates which type of conflict? a. Organizational b. Intrapersonal c. Interpersonal d. Disruptive

ANS: C Interpersonal conflict transpires between and among nurses, physicians, members of other departments, and patients.

22. Lee, the head nurse in ER, has attempted to meet Jillian, one of her staff RNs, for several days to discuss concerns about Jillian's relationships with her team members. Lee hopes to offer Jillian coaching so that Jillian's relationships can be more satisfying for Jillian and her team members. Each time Lee and Jillian set a time to meet, Jillian phones in sick. In this situation, Lee and Jillian are demonstrating: a. similar conflict management strategies. b. escalation of conflict. c. avoidance and compromise strategies. d. competing and compromise strategies.

ANS: C Jillian is demonstrating avoidance by staying away from meetings to discuss her team relationships, and Lee is demonstrating compromise by offering coaching in return for Jillian's being able to engage in more satisfying relationships.

17. Kala, a unit manager, in discussing a role the CEO would like her to perform, makes the following statement, "I will sit on the hospital task force on improving morale if you send me to the hospital's leadership training classes next week, so I can further develop my skills and thus be more effective." Which of the following conflict management styles is Kala using? a. Collaborating b. Avoiding c. Negotiating d. Accommodating

ANS: C Negotiation involves an exchange of concessions (membership on a committee in return for attendance at a workshop) or trading. This strategy supports a balance of power.

12. Which of the following exemplifies accountability? Karen, the nurse manager on 5E: a. consistently submits her budgets on time. b. gets along well with her staff and with other managers. c. outlines her rationale for reduction of RN coverage on nights to the Nursing Practice Committee after serious patient injury. d. actively solicits ideas regarding scheduling from her staff.

ANS: C Reliability, dependability, and obligation to fulfill the roles and responsibilities of the nurse manager are consistent with responsibility. Accountability refers to being answerable for actions and results.

The oncology clinic manager and the educational coordinator asked nursing staff to complete a brief written survey to assess their attitudes and knowledge related to having used the new infusion equipment for 6 weeks. The stage of change in this situation is: a. Developing awareness. b. Experiencing the change. c. Integrating the change. d. Perceiving awareness.

ANS: C This particular initiative assesses the success with which the change has been integrated into everyday practice after it has been experienced, or the degree to which staff members have accepted using the new infusion equipment.

6. To solve a problem, the nurse manager understands that the most important problem-solving step is: a. the implementation phase. b. identification of numerous solutions. c. accurate identification of the problem. d. evaluation of the effectiveness of problem resolution.

ANS: C To proceed effectively, it is important to determine if a problem exists and to accurately identify a problem. Failure to resolve problems is most often linked to improper identification of the problem.

To effectively delegate in a team nursing environment, the RN team leader must be familiar with the legal and organizational roles of each group of personnel and must: a. be able to effectively communicate with patients. b. build relationships with physicians. c. be able to adapt to daily changes in staffing. d. adapt in communicating information to her supervisor.

ANS: C A particular challenge in team nursing is that staff mixes and staff may change daily because of individual schedules and shortages.

A factor evaluation system: a. utilizes financial data to determine number of staff-to-patient ratios. b. utilizes DRGs to determine acuity on a unit. c. combines interventions and time required for interventions to determine levels of care required. d. combines financial resources and nursing interventions to determine patient contact hours.

ANS: C A factor evaluation system considers tasks, thought processes, and patient care activities and gives them a time or rating. These are then used to determine the number of patient care hours required.

A nurse manager has been employed in the same facility for 20 years and has held the same position. This career style is known as: a. linear. b. spiral. c. steady state. d. entrepreneurial/transient.

ANS: C Career styles that are marked by selecting and staying in a role throughout a career are characterized as steady state.

On your curriculum vitae, which of the following is the recommended approach for listing employment and educational history information? a. 1979 RN Diploma 1985 BScN 2002 MN b. 2002 Mount Rush Health Center Staff Nurse 1997 Cedar Falls Clinic Staff Nurse 2007 Kilkarney Rehab Center Case Manager c. 2007 MN 2005 BScN d. 1997 Sturgeon County Hospital Head Nurse 2002 Sturgeon County Supervisor 2007 Sturgeon County Director

ANS: C Information that is included in the body of the curriculum vitae should always be in reverse chronological order so that the most recent and, presumably, most relevant job information appears first.

To effectively achieve a change goal/outcome in a change situation, the wound care specialist will: a. Preserve the status quo. b. Diminish facilitators and reinforce barriers. c. Weigh the strength of forces. d. Strengthen facilitating forces.

ANS: D For change to be effective, the facilitators must exceed the force of the barriers; thus, strengthening the facilitating forces would achieve this aim.

Resistance is most likely when change: a. Is not well understood. b. Involves many layers in an organization. c. Involves nonprofessional workers. d. Threatens personal security.

ANS: D Resistance and reluctance commonly occur when personal security is threatened and may involve loss of confidence in abilities or loss of job or financial security.

To achieve nurse-sensitive care standards developed by the NQF, you advocate for which of the following in your health facility? a. Evidence-based practice to reduce the prevalence of pressure sores b. Implementation of informatics at the bedside c. Staff-manager conferences to review reporting of adverse medical events d. Patient councils to review food, recreation, and nurse-patient relations

a. Evidence-based practice to reduce the prevalence of pressure sores The National Quality Forum (NQF) outlines nursing-centered intervention measures related to prevalence of pressure sores, ventilator-associated pneumonias, volunteer turnover, nursing care hours per day, and skill mix of staff.

A dispute arises between an RN staff member and an LPN over a patient issue. The tension between the two begins to affect other staff members, who are drawn into the conflict; eventually, the team becomes polarized toward either the RN or the LPN. This situation might have been prevented through: a. Expediency in responding to the initial dispute, once it became apparent that it could not be resolved by the two parties themselves. b. Asking other staff members what the real issues were in the dispute between the RN and the LPN. c. Reassigning one of the parties to another unit when it became apparent that the two individuals could not resolve the dispute themselves. d. Calling a staff meeting at the onset of the dispute to allow the team and the RN and LPN to discuss the initial dispute.

a. Expediency in responding to the initial dispute, once it became apparent that it could not be resolved by the two parties themselves. The initial step in conflict resolution should have involved an expedient response to the issues and putting a focus on the issues involved in the dispute between the LPN and RN through negotiation involving the two parties, before the dispute involved others.

As a leader on a rehab unit, you encourage all staff members to see themselves as having a role in decision making and quality care. You see your role as involving particular responsibilities in decision making but not as a hierarchal role. This view of decision making and leadership is consistent with: a. Trait theories. b. Complexity theory. c. Situated theory. d. Emotional intelligence.

b. Complexity theory. Complexity theory involves envisioning each member of the team involved in decision making, management, and leadership, with the leader not seen in a hierarchal relationship to other team members.

As the nurse manager who wants to increase motivation by providing motivating factors, which action would you select? a. Collaborate with the human resource/personnel department to develop on-site daycare services. b. Provide a hierarchical organizational structure. c. Implement a model of shared governance. d. Promote the development of a flexible benefits package.

c. Implement a model of shared governance. Complexity theory suggests that systems interact and adapt and that decision making occurs throughout systems, as opposed to being held in a hierarchy. In complexity theory, every voice counts, and therefore all levels of staff would be involved in decision making. This principle is the foundation of shared governance.

Mr. M. complains to you that one of your staff asked him details about his sexual relationships and financial affairs. He says that these questions were probing and unnecessary to his care, but he felt that if he refused to answer, the nurse would be angry with him and would not provide him with good care. Mr. M.'s statements reflect concern with: a. Privacy. b. Confidentiality. c. Veracity. d. Informed consent.

ANS: A Privacy protection includes protection against unwarranted intrusion into the patient's affairs.

After a major flu vaccination campaign, an agency bills a private insurance company for allowable costs for administration of each vaccination according to the schedule established by the insurance company for reimbursement. This is an example of which major payment method? a. Cost-based reimbursement b. Charges c. Contractual allowance d. Prospective reimbursement

ANS: A Because the agency is submitting costs after the campaign is completed and in accordance with an established schedule of allowable costs, a retrospective, cost-based reimbursement payment method is being utilized. If a desire for profit was indicated, then the answer would be "charges."

Case managed care may enhance profit in a for-profit health organization by: a. minimizing costs in high resource consumption areas. b. combining licensed and non-licensed care providers in delivering patient care. c. increasing reimbursement from third-party payers. d. reducing the amount of technology used to support clinical decision making.

ANS: A Case managed care is not revenue generating but rather revenue protecting in that better coordination of care enables efficient achievement of patient outcomes, can result in shorter length of stay, and can prevent readmission.

In comparing team and functional models of care, a nurse manager favors the team model. In particular, she finds that the team model: a. can be effective in recognizing individual strengths and backgrounds of staff. b. promotes autonomy and independence for the RN. c. avoids conflict because of role clarity. d. is efficient in delivering care to a large group of patients, utilizing a staffing mix.

ANS: A Team nursing delivers care to a small group of patients, using a mix of licensed and unlicensed personnel. Team nursing uses the strengths of each caregiver.

A new graduate is seeking a new position in nursing and wants to "sell" herself effectively. The best strategy is to: a. create a résumé. b. practice interviewing. c. call the personnel offices. d. create a curriculum vitae.

ANS: A A résumé provides a customized overview of details of your professional background that relates specifically to a position for which you are applying. Résumés advertise your skills to a prospective employer.

In reviewing the job description of a nurse manager, the staff becomes aware that a nurse manager's role is complex. Which of the following duties are required of a nurse manager? (Select all that apply.) a. Ensure unit productivity reports. b. Develop policy and legislation to protect nurses' well-being. c. Plan staffing of UNPs only. d. Prepare a unit budget that reflects unit staffing needs. e. Monitor nurse-sensitive indicators such as falls and incidence of infections

ANS: A, D, E A nurse manager manages financial resources by developing business and staffing plans.

11. With delegation, responsibility and accountability remain with the: a. physician. b. professional who delegates. c. individual who receives the delegation. d. individual who previously performed the task.

ANS: B Even though the delegatee (the one who receives the delegation) receives direction from the professional who delegates a task and must have the authority to complete it, the delegator retains accountability for the overall outcome and completion of the activity. The delegatee has responsibility (obligation to engage in the task) and authority for the task.

A staff nurse regularly works two 12-hour shifts each week and one 8-hour shift every other week. How many FTEs is this position? a. 0.6 b. 0.7 c. 0.8 d. 1

ANS: B Assuming that a full-time RN works 2080 hours/year (40 hours x 52 weeks), the nurse works (24 hours x 52 weeks + 8 hours x 26 weeks)/2080, which is 0.7 FTE.

A patient refuses a simple procedure that you believe is in the patient's best interest. The two ethical principles that are directly in conflict in such a situation are: a. Fidelity and justice. b. Veracity and fidelity. c. Autonomy and beneficence. d. Paternalism and respect for others.

ANS: C Autonomy refers to the freedom to make a choice (e.g., refuse a procedure), and beneficence to doing good (performing a procedure that will benefit the patient).

On your nursing unit, you employ LPNs, RNs, and advanced practice nurses. You will need to be familiar with at least: a. Two nursing practice acts. b. Two nursing practice acts in most states. c. At least one nursing practice act. d. One nursing practice act and a medical act.

ANS: C In all states, you will need to be familiar with at least one nursing practice act. In some states, there may be two nursing practice acts if RNs and LPNs/LVNs come under different licensing boards.

A staff nurse in the area that you manage has excelled in the delivery of patient education. You are considering implementing a new job description that would broaden her opportunity to teach patients and orient new staff members to the value of patient education. The ethical principle that you are most directly reinforcing is: a. Justice. b. Fidelity. c. Paternalism. d. Respect for others.

ANS: C The principle of paternalism allows one person to make partial decisions for another and is most frequently deemed to be a negative or undesirable principle. Paternalism, however, may be used to assist persons to make decisions when they do not have sufficient data or expertise. Paternalism becomes undesirable when the entire decision is taken away from the employee. justice: treating all persons equally and fairly fidelity: keeping one's promises or commitments, respect for others: highest principle. respect for others acknowledges the right of individuals to make decisions and to live by these decisions.

2. The charge nurse is making patient assignments for the next shift on the unit. There is one critical patient on the unit, who is going to require more care than the other patients. Before delegating this patient in an assignment, what is the appropriate action by the charge nurse? a. Delegate the admission assessment to the LPN. b. Review the employee's performance assessment for the most recent period. c. Assess the amount of guidance and support needed for the nursing care of the patient. d. Create a task analysis of critical behaviors for the individual.

ANS: C To delegate effectively, the charge nurse must assess the abilities required in the situation and the abilities that staff have to anticipate the amount of direction, monitoring, explanation, and independence that can be assumed.

The home health agency hired an expert in financial management to evaluate and propose a plan for reversing growing expenses and decreasing revenues. The expert is well respected, both personally and professionally, by members living in this small community. To be effective, staff will need to perceive this change agent as: a. Trusted, quiet. b. Flexible, informal. c. Credible, enthusiastic d. Communicative, personable.

ANS: C To influence the decision, the expert must be seen as having knowledge of what matters to the people that they lead and of the change area itself. The expert also must be enthusiastic and communicative and have referent power.

1. You are a member of a team assigned to care for 15 general medical/surgical clients. You have all worked well together in the past in this same type of care. If you are assigned to coordinate this team's work, your best strategy, based on the Hersey model, would be to: a. have a list of tasks to be accomplished and tell each member of the team what he or she must do. b. encourage people to discuss their frustrations in providing this care. c. ignore them—they've done it before. d. provide minimal direction and let them come to you with questions.

ANS: D According to the Hersey model, when ability (skills, job knowledge) and willingness are strong, the role of the delegator is less ("delegating behavior").

When hiring a case manager for a rehabilitation setting, you would most likely consider a: a. registered nurse with a master's degree. b. physiotherapist with a background in stroke rehabilitation. c. social worker with a background in counseling. d. health professional with advanced background who is client and outcome focused.

ANS: D Case managers can come from a variety of disciplines but should have advanced preparation with the particular at-risk population being served, be comfortable in an advocate role, and be outcome and patient focused.

In the past year, you have noticed an increase in patient falls on your unit. In reading studies related to staffing and patient outcomes, you realize that you will need to plan for: a. higher patient care hours. b. safer facilities. c. institution of a patient-classification system. d. an increased number of RN positions.

ANS: D Lower fall rates are shown to be related to higher total nursing hours and a higher percentage of nursing hours supplied by RNs.

Which of the following needs revision on a résumé or CV? a. John Jones 87 Highway Drive City, MI 79110 [email protected] b. M. Howes Anyway Highway City, MO 77700 (H) 777-777-0000 e-mail: [email protected] c. Dr. L. Jones 99 Carway Drive City, NY 84003 (H) 999-999-0000 (Cell) 999-000-9999 d. Tanya Jones 67 Honeywell Drive City, MO 66907 [email protected]

ANS: D When including Web or e-mail addresses, it is important to use addresses that are not overly casual or that communicate personal information.

Which leadership role is associated with advocacy? A) Create a climate where advocacy and its associated risk-taking are valued B) Give subordinates and patients adequate information to make informed decisions C) Ensure that rights and values of patients supersede those of the health-care providers D) Seek appropriate consultation when advocacy results in intrapersonal or interpersonal conflict

Ans: A Feedback: A leadership role associated with advocacy is to create a climate where advocacy and its associated risk-taking are valued. Management functions include the following: give subordinates and patients adequate information to make informed decisions; ensure that rights and values of patients supersede those of the health-care providers; and seek appropriate consultation when advocacy results in intrapersonal or interpersonal conflict.

22. Which strategy demonstrates an understanding of the management's role as advocate for their subordinates? Select all that apply. A) Staff representatives on all unit committees B) Monthly ìbrown bag lunchesî with the staff C) Requests revising criteria for granting a ìleave of absenceî D) Working with primary care providers to expand standing nursing orders

Ans: A, B, C, D Feedback: The following are suggestions for creating an environment that promotes subordinate advocacy: Invite collaborative decision making; get to know staff personally; ìgo to batî for staff when needed; and promote nurse autonomy. Rather than anticipating educational needs, the manager advocates by asking for and then respecting their suggestions by planning accordingly.

24. A nurse-manager has been asked to write an article for the newspaper addressing a health-care issue of local concern. Which statement demonstrates the nurse understandings the appropriate way to response to such a request? Select all that apply. A) "I'll include the latest data on the subject." B) "When will you need to have the final draft of my article?" C) "Are you comfortable with me focusing on 3 key points related to this topic?" D) "That isn't my field of expertise but I will contact a peer with that knowledge."

Ans: A, B, C, D Feedback: Tips of effective interaction with the media include respecting and meeting their deadlines, having key facts ready to include, limiting key points to two or three, and not being afraid to say you lack that specific expertise. One should assume, until proven otherwise, that the reporter will be fair and accurate in his/her reporting.

Which of the following accurately represents the concept of political activism? a. Meghan, an ER supervisor, encourages staff to write letters to the local health board, protesting closure of the ER and loss of 30 full-time jobs. b. Sarah refuses involvement in her professional organization but is heavily involved in the Little League organization to which her son belongs. c. Because of her influential contributions to position papers on health care, Roberta is asked to let her name stand for election as chair of the local organization of gerontology nurse practitioners, who are lobbying for increased certification standards. d. Sondra volunteers to run for office in her state nursing organization because of her concern about the underrepresentation of expertise from her area of nursing practice.

D

4. Which statement about a national Patient Bill of Rights is accurate? A) Not yet been enacted by the federal government B) Became the law of the land C) Became legally binding in every state D) Has not been beneficial to patients

Ans: A Feedback: Although there has been significant progress in the field of patient rights since 1960, there is still no comprehensive federal legislation directed at the granting and protection of patient rights.

Patient perceptions are useful in: a.Determining disciplinary actions in QI. b.Establishing the competitive advantage of QI decisions. c.Providing one source of data for QI initiatives. d.Establishing blame for poor-quality care.

Assisting to establish priorities among possible changes to care identified in QI

Nurses who engage in in-fighting, seek physician support against nursing colleagues, and avoid membership in nursing organizations: a. Refuse to believe that they are acting like members of groups that suffer socioeconomic oppression. b. Do not understand how their failure to exercise power can limit the power of the whole profession. c. Purposefully choose to exercise their power in the workplace through indirect means. d. Suffer from learned helplessness as a result of abuse by powerful nurse executives.

B

A nurse manager recognizes the need to expand her professional network as she begins a job search for a middle-management position. Which of the following actions is least likely to expand her job-searching network? a. Reviewing her address book or card file for names and phone numbers of former colleagues who are now in middle-management positions b. Making an appointment to meet with a former instructor from her graduate program in nursing administration c. Making a long overdue return call to a former colleague who is now a chief nurse executive d. Attending a state-level conference for nurse managers and executives and attending informal luncheons and receptions

C

Politics is usually: a. Confined to legislatures. b. Seen in dysfunctional workplaces. c. Found in all social organizations. d. A representation of self-interest.

C

An example of an effective patient outcome statement is: a.Eighty percent of all patients admitted to the Emergency Department will be seen by a nurse practitioner within 3 hours of presentation in the Emergency Department. b.Patients with cardiac diagnoses will be referred to cardiac rehabilitation programs. c.The hospital will reduce costs by 3% through the annual budget process. d.Quality is a desired element in patient transactions

Eighty percent of all patients admitted to the Emergency Department will be seen by a nurse. Practitioner within 3 hours of presentation in the Emergency Department

Learning organization

Entities that emphasize flexibility and responsiveness. The designation of a type of organization in which continual learning as an expectation permeates all levels to promote adequate responses required by dynamic, accelerated change.

The SBAR approach to patient safety encourages: a. Consistency in assessment and practices. b. Continuing education. c. Multidisciplinary approaches. d. Patient feedback.

a. Consistency in assessment and practices. The use of SBAR (Situation, Background, Assessment, and Recommendation) checklists are designed to decrease omission of important information and practices.

Nurses were long viewed by physicians, legislators, the media, and others as powerless because nurses a. declined to participate in political activities in the earliest years of the profession in the United States. b. as women were subject to control by medicine and other paternalistic groups; women had limited legal rights in late 19th-century America. c. and nursing leaders in the mid-20th century did not wish to be viewed by those outside of nursing as pushy or demanding. d. were subject to nursing practice acts that limited their ability to take political action.

as women were subject to control by medicine and other paternalistic groups; women had limited legal rights in late 19th-century America.

As the RN charge nurse on the night shift in a small long-term care facility, you've found that there is little turnover among your LPN and nursing assistant (NA) staff members, but they are not very motivated to go beyond their job descriptions in their work. Which of the following strategies might motivate the staff and lead to greater job satisfaction? a. Ask the director of nursing to offer higher wages and bonuses for extra work for the night LPNs and NAs. b. Allow the LPNs and NAs greater decision-making power within the scope of their positions in the institution. c. Help the LPNs and NAs with their work, whenever possible. d. Ask the director of nursing to increase job security for night staff by having them sign contracts that guarantee work.

b. Allow the LPNs and NAs greater decision-making power within the scope of their positions in the institution. Hygiene factors such as salary, working conditions, and security are consistent with Herzberg's two-factor theory of motivation; meeting these needs avoids job dissatisfaction. Motivator factors such as recognition and satisfaction with work promote a satisfying and enriched work environment. Transformational leaders use motivator factors liberally to inspire work performance and increase job satisfaction

Joan, the nursing unit manager, finds it difficult to work with Thomas, a new graduate. Thomas has many ideas, and his manner of presenting them irks Joan. After reflection and discussion with others, Joan recognizes that she also feels threatened by his behavior. She comes to understand that Thomas is trying to establish his own role on the unit, is not trying to challenge her, and needs guidance, coaching, and affirmation. Joan is demonstrating: a. A positive self-concept. b. Deepening self-awareness. c. Leadership. d. Acquiescence.

b. Deepening self-awareness. According to Goleman (1995), stepping outside oneself to envision the situation while assuming ownership is a component of emotional intelligence.

A nurse manager of a 20-bed medical unit finds that 80% of the patients are older adults. She is asked to assess and adapt the unit to better meet the unique needs of the older adult patient. Using complexity principles, what would be the best approach to take in making this change? a. Leverage the hierarchical management position to get unit staff involved in assessment and planning. b. Engage involved staff at all levels in the decision-making process. c. Focus the assessment on the unit, and omit the hospital and community environment. d. Hire a geriatric specialist to oversee and control the project.

b. Engage involved staff at all levels in the decision-making process. Complexity theory suggests that systems interact and adapt and that decision making occurs throughout the systems, as opposed to being held in a hierarchy. In complexity theory, every voice counts, and therefore, all levels of staff would be involved in decision making.

The NQF provides a model for advancement of healthcare quality that could be used in healthcare organizations. The use of this model by the Centers for Medicare & Medicaid Services specifically affects the interaction between adverse events and: a. Staffing. b. Funding. c. Composition of executive councils. d. Composition of consumer-based councils.

b. Funding. The Centers for Medicare & Medicaid Services (CMS) have adopted a policy based on the NQF's "never events." The CMS will no longer pay for patient conditions or events that result from poor practice while patients are under the care of a health professional

A nurse manager must implement a 2% budget cut on the nursing unit. Which approach should the manager use to most effectively empower the staff of the unit? a. Discuss the guidelines for the budget cuts with the staff, making the decisions with those who participate. b. Inform the staff of the budget cuts in a series of small group meetings, and accept their ideas in writing only. c. Provide the staff with handouts about the budget cuts, and let them make recommendations in writing. d. Hold a series of mandatory meetings on the budget cuts, asking staff for ideas on the cuts.

A

3. The nurse manager is aware that conflict is occurring on her unit; however, she is focused on preparing for a state health department visit, so she ignores the problem. What factor can increase stress and escalate conflict? a. The use of avoidance b. An enhanced nursing work force c. Accepting that some conflict is normal d. Managing the effects of fatigue and error

ANS: A Avoidance as a conflict-management style prolongs conflict and tends to escalate conflict.

Three gravely ill patients are candidates for the only available bed in the ICU. As the supervisor, you assign the bed to the patient with the best chance of recovery. This decision reflects which of the following ethical principles? a. Beneficence b. Autonomy c. Veracity d. Nonmaleficence

ANS: A Beneficence refers to doing what's good for the patient; in this situation, doing what's good means providing care to the patient with the best likelihood of recovery.

Examples of sentinel events include (select all that apply): a.Forceps left in an abdominal cavity. b.Patient fall, with injury. c.Short staffing. d.Administration of morphine overdose. e.Death of patient related to postpartum hemorrhage.

ABDE

Amy has worked in the dialysis unit on staff for about 12 years. She is frequently consulted by other nursing staff regarding protocols and policies on the unit. What type of power is Amy using? a. Position power b. Expert power c. Personal power d. Competency power

B

Which of the generations is challenged by sharing leadership with other generations? a. Baby Boomers b. Generation Y c. Generation X d. Generation Z

Baby Boomers

A manager relies on his director (immediate supervisor) for advice about enrolling in graduate school to prepare for a career as a nurse executive. The director may exercise what kinds of power in the relationship with the manager in this advisory situation? a. Expert, coercive, and referent b. Reward, connection, and information c. Referent, expert, and information d. Reward, referent, and information

C

Despite repeated invitations by his colleagues to become involved in regional and state nursing practice committees, Tom refuses. His reason is that "nursing committees rarely get anything worthwhile done because of politics and conflicts." According to the text, Tom's view of involvement: a. Is rare in nursing today. b. Reflects a fear of power. c. Reflects the essential process of power. d. Reflects empowerment and capacity to make his own decisions.

C

A method commonly used in Quality Assurance to monitor adherence to established standards is: a.A Pareto chart. b.Brainstorming. c.Patient interviews. d.Chart audit.

Chart audit

Unplanned change

Disconcerting, unanticipated, adaptive change.

Facilitators

Favors change. Factors, internal or external to the change situation that promote movement towards desired outcome.

A nursing unit is interested in refining its self-medication processes. In beginning this process, the team is interested in how frequently errors occur with different patients. To assist with visualizing this question, which organizational tool is most appropriate? a.Histogram b.Flowchart c.Fishbone diagram d.Pareto char

Histogram

The outcome statement "Patients will experience a ten percent reduction in urinary tract infections as a result of enhanced staff training related to catheterization and prompted voiding" is: a.Physician-sensitive and nonmeasurable. b.Measurable and nursing-sensitive. c.Precise, measurable, and physician-sensitive. d.Patient care-centered and nonmeasurable

Measurable and nursing-sensitive

A nursing-led classification system that has led to greater reliability and standardization in data utilized for QI processes is: a. NANDA. b. AHRQ. c. NIOSH. d. Nursing process.

NANDA

In determining the relationship between injury-producing falls and proposed preventive measures as part of the QI process, a QI team might turn to which of the following for confirmatory evidence? a.NDNQI b.NANDA c.NIOSH d.AHRQ

NDNQI

Before beginning a continuous quality improvement project, a nurse should determine the minimal safety level of care by referring to the:

Nursing care standards

As a nurse manager, you know that the satisfaction of patients is critical in making QI decisions. You propose to circulate a questionnaire to discharged patients, asking about their experiences on your unit. Your supervisor cautions you to also consider other sources of data for decisions because: a.The return rate on patient questionnaires is frequently low. b.Patients are rarely reliable sources about their own hospital experiences. c.Hospital experiences are frequently obscured by pain, analgesics, and other factors affecting awareness. d.Patients are reliable sources about their own experiences but are limited in their ability to gauge clinical competence of staff.

Patients are reliable sources about their own experiences but are limited in their ability to gauge clinical competence of staff

Change process

The series of ongoing efforts applied to managing a change.

Chaos theory

Theoretical construct defining the random-appearing yet deterministic characteristics of complex organizations.

A grievance brought by a staff nurse against the unit manager requires mediation. At the first mediation session, the staff nurse repeatedly calls the unit manager's actions unfair, and the unit manager continues to reiterate the reasons for her actions. What would be the best course of action at this time? a. Send the two disputants away to reach their own resolution. b. Involve another staff nurse in the discussion so as to clarify issues. c. Ask each party to examine her own motives and issues in the conflict. d. Continue to listen as the parties repeat their thoughts and feelings about the conflict.

c. Ask each party to examine her own motives and issues in the conflict. Ury, Brett, and Goldberg outline steps to restoring unity, the first of which is to address the interests and involvement of participants in the conflict by examining the real issues of all parties.

In designing a quality, safe healthcare environment, the primary emphasis needs to be on: a. Evidence-based practice. b. Informatics. c. Staffing. d. The patient.

d. The patient. Focusing on the patient moves care from concern about who controls care to a focus on what care is provided to and with patients, which was an aim identified in the IOM report Crossing the Quality Chasm.

Barriers

Against change. Factors, internal or external to the change situation, that interfere with movement towards a desirable outcome.

4. During a fire drill, several psychiatric patients become agitated. The nurse manager quickly assigns a staff member to each patient. This autocratic decision style is most appropriate for: a. routine problems. b. crisis situations. c. managers who prefer a "telling" style. d. followers who cannot agree on a solution.

ANS: B An autocratic style is appropriate when rapid decision making is required, such as in a crisis situation.

An example of one strategy used to improve participation in the change process by staff fitting the behavioral description of innovators and early adopters is to: a. Repeat the benefits of the change. b. Share change experiences early in the process. c. Initiate frequent interactions among staff. d. Provide select information to the staff.

ANS: B Connecting innovators and early adopters to new ideas and with new peers keeps them at the cutting edge.

The SBAR system of communications is one of the most used communication systems in health care because: a. it deals with all aspects of communications in patient care except communication with the physician. b. the nurse is on the same communication level as administration. c. it honors a familiar, structured transfer of information among health professionals. d. it honors an unstructured transfer of information.

ANS: C The SBAR system was developed by professionals in the California Kaiser Permanente System and involves direct, respectful communication skills among professionals with the aim of quality patient care.

2. The nurse manager of a rehab unit wants to purchase a new antiembolic stocking for use with patients. To make a high-quality decision, the nurse manager would: a. involve the rehab staff in the decision. b. involve the sales representative. c. make the decision alone. d. involve administration in the decision.

ANS: A In a shared decision model, the decisions are made through an interactive, deliberate process and the staff may express and discuss options and preferences. The shared decision model has been shown to increase work performance and productivity, decrease employee turnover, and enhance employee satisfaction.

To engage your staff in awareness of their current practice and how it is affirmed or not by evidence, you plan a short series of learning presentations on evidence and use of heparin and saline to maintain IV patency. You meet with the educator to plan out the goals for each session with the overall purpose of increasing knowledge and awareness of staff in readiness to consider questions related to the IV practice. This learning approach is an example of which change management approach? a. Linear b. First-order c. Facilitative d. Integrative

ANS: A In the second stage, the moving or changing stage of Lewin's first-order, planned change process, planned interventions and strategies are executed to support the implementation of the change. One commonly used method is educating staff about the need for the change.

Elizabeth, an RN with approximately 15 years of service on your unit, walks away from one of the learning sessions on IV care and you overhear her telling a colleague that she thought the session was a waste of time because "the unit has been using heparin for years and there has never been any adverse effects." You follow up with Elizabeth and discover that she is really quite angry about the information sessions because she feels that you are implying that "what she has been doing all these years means that she is incompetent and doesn't care about her patients." After speaking with Elizabeth a few days later you discover that she is now fine with the change but is concerned that other areas of the organization might resist the change because of perceptions related to patient safety and cost. She suggests that it is important to bring pharmacy on board as they have had previous concerns about the use of heparin. In relation to change theory, this is indicative of: a. Systems level thinking. b. Linear thinking. c. Interprofessional collaboration. d. First-order change.

ANS: A Senge's complexity theory, Bevan's Seven Change Factors, and general systems theory all highlight connectivity and the idea that changes are not isolated events.

The mediator noticed that tension was still evident between the nurse manager and staff members. He informed the chief nursing officer that to begin team building, it would be important that everyone: a. work together in a respectful, civil manner. b. use avoidance techniques when confronted with a conflict. c. develop a personal friendship with each other. d. socialize frequently outside of work.

ANS: A For team building to occur, team members need to be able to listen actively and respect one another's opinions, while feeling comfortable in openly expressing their own.

A mediator suggested that the nurse manager and staff members decide on a method to resolve conflicts. It is important to have agreements about how team members will work together because: a. if there are no agreements, each member will make up rules about how to handle disagreements and relationships. b. people are naturally difficult and will not work well together without such agreements. c. people will naturally ask for agreements about how to be together. d. a way to eliminate nonproductive team members must be available.

ANS: A People must agree on the goals and mission with which they are involved. They have to reach some understanding of how they will exist together. Tenets or agreements such as "I will respectfully speak promptly with any team member with whom I have a problem" go a long way to avoid gossiping, backbiting, bickering, and misinterpreting others. Without agreement, people have implicit permission to behave in any manner they choose toward one another, including angry, hostile, hurtful, and acting-out behavior.

Trust is an important aspect of helping relationships, therapeutic communications, and the positive communications model. Which statement does not involve or define trust? Trust: a. involves decisions to manipulate situations to gain advantage over another. b. is the basis by which leaders facilitate the activities and progress of a team. c. is low among members and leaders in poorly performing teams. d. involves what we say and not necessarily what we do.

ANS: A Trust is high in high-performing teams and involves not consciously taking advantage of others and behaving in a way that inspires trust. It is the basis by which leaders facilitate the activities and progress of a team.

A busy neurologic ICU and step-down unit most likely would use which patient-classification system? a. Factor evaluation b. Prototype evaluation c. Hybrid system d. AHRQ system

ANS: A A factor evaluation system is considered more objective than a prototype evaluation system. It gives each task, thought process, and patient care activity a time or rating. Some patient types with a single healthcare focus, such as maternal deliveries or outpatient surgical patients, would be appropriately classified with a prototype system. Patients with more complex care needs and a less predictable disease course, such as those with pneumonia or stroke, are more appropriately evaluated with a factor system.

The difference between staffing and scheduling is that staffing: a. puts the right person in the right position. b. puts the right person in the right time and place. c. refers to the number of nursing hours per patient per day. d. looks after interpretation of benefits and compensation.

ANS: A Nursing staffing involves planning for hiring and deploying qualified human resources to meet the needs of a group of patients. Scheduling, on the other hand, is a function of implementing the staffing plan by assigning unit personnel to work specific hours and days of the week.

A nurse manager must also consider a number of internal variables that will affect staffing patterns. An internal variable to be considered is: a. organizational staffing policies. b. state licensing standards. c. American Nurses Association. d. consumer expectations.

ANS: A State licensing standards outline what a nurse can do. Internal policies determine what a nurse may do in a particular setting as well as the amount of flexibility that is allowed to manage times of high and low volumes, as well as changes in acuity. Organizational policies can put the nurse manager in a situation where patient safety cannot be maintained or financial obligations met.

1. In a busy rehabilitation unit, the team manager decided that the best way to reward the staff was to give them a monetary bonus rather than time off. The staff was very concerned about the decision and went to the administration with a number of complaints. Critical thinking is a process that entails a number of steps. What steps did the manager omit? She should have:(Select all that apply.) a. identified what assumptions were underpinning the issues. b. considered why it was important to make this change or the context for the change. c. considered how this change might affect staff relationships. d. attained a majority consensus of all staff.

ANS: A, B, C Taking a majority consensus is not a step in the critical thinking process. Points A, B, and C are "what," "why," and "how" questions that are part of effective critical thinking processes.

Which of the following are examples of application of the Leadership Rounding Tool? (Select all that apply.) a. "What is working well for you during bedside reporting?" b. "What has not worked for you today?" c. "Is there someone on your team who deserves special recognition for her efforts in the implementation?" d. "Did you have a good vacation?"

ANS: A, B, C, D The Leadership Rounding Tool suggests establishing and maintaining rapport and asking what is working well, what was a barrier, and who should be recognized, as well as answering tough questions.

Mobilizing others to accomplish extraordinary things requires what leadership behaviors? (Select all that apply.) a. Celebrating the successes of others b. Demonstrating exceptional technical skills c. Imagining possibilities d. Establishing a sense of "being in this together"

ANS: A, C, D Leaders who inspire teams to accomplish extraordinary things or to display synergy model the way, inspire shared vision, challenge the status quo, and encourage the heart by celebration of success.

20. John Smith, one of three managers at BSG Labs, drafted a policy that would allow his department to do more testing in his lab. This policy included the times for regular collection as well as a new process for emergency laboratory testing. The policy and procedures were never followed. The reason was that: a. the policy was too lengthy and inundated readers with too much detail. b. the policy made decisions for other departments in the company. c. the staff did not believe that the new policy would be effective. d. testing should not be done in the lab.

ANS: B Two primary criteria make for effective decisions. First, the decision must be of a high quality; that is, it achieves the predefined goals, objectives, and outcomes. Second, those who are responsible for its implementation must accept the decision. Higher quality decisions are more likely to result if groups are involved in the decision-making and problem-solving process. Taking ownership of the process and outcome provides a smoother transition in changes.

You are charged with developing a new nursing curriculum and are committed to developing a curriculum that reflects the needs of the profession and of the workplace. To address deficits that may already be present in nursing curricula related to the workplace, you include more content and skills development related to: a. therapeutic communication with patients. b. effective communication in the workplace. c. increased emphasis on sender-receiver dyads. d. generational differences in communication.

ANS: B Nursing programs teach therapeutic communications with patients and their families. Little focus, however, is placed on effective communication in the workplace, although communication is essential to building and maintaining smoothly functioning teams.

A small rural hospital has been designated as a critical access hospital. It has 40 beds and an average occupancy of 34 beds. To prepare the staffing, the chief nursing officer computes the occupancy as being: a. 90%. b. 85%. c. 75%. d. 60%.

ANS: B A way to assess a unit's activity level is to calculate the percentage of occupancy. Formula: daily patient census (rounded) divided by the number of beds in the unit The occupancy level is calculated as 34/40, or 85%.

A nurse manager uses many sources of data when planning the unit's workload for the year. Which of the following data must be considered in the planning? a. Hours of operation of the unit b. Trends in acuity on the unit c. Maximum work stretch for each employee d. Weekend requirements

ANS: B Acuity levels are determined through classification systems, which determine the nursing resources required.

Staff members on your unit raise concern that there is rising acuity on the unit and lack of responsiveness in addressing these needs through appropriate staffing. They point to increased incidences of adverse and sentinel events on the unit. To address this concern, your hospital organization would do best to: a. implement a patient-classification system immediately. b. participate in databases that compare the outcomes and staffing levels versus those of similar institutions. c. provide increased numbers of staff to the unit. d. ignore such concerns because acuity is variable.

ANS: B Staff morale suffers both when acuity models indicate a gap between staffing and acuity and when there is no model but perceived acuity that is not being addressed. A truer approach is to monitor patient outcomes and participate in national databases that measure staffing levels through comparison with like institutions.

3. Several nurses on an adolescent psychiatric unit complain that the teens are becoming unmanageable on the 0700-1900 shift. To resolve this problem, the nurse manager decides that the staff should have a brainstorming session. The goal of brainstorming is to: a. evaluate problem solutions. b. critique the ideas of others. c. generate as many solutions as possible. d. identify only practical and realistic ideas.

ANS: C Brainstorming encourages creativity when one is beginning to problem-solve and avoids premature shutting down of ideas through early evaluation. The goal is to generate ideas, no matter how seemingly unrealistic or absurd.

Elizabeth, an RN with approximately 15 years of service on your unit, walks away from one of the learning sessions on IV care and you overhear her telling a colleague that she thought the session was a waste of time because "the unit has been using heparin for years and there has never been any adverse effects." You follow up with Elizabeth and discover that she is really quite angry about the information sessions because she feels that you are implying that "what she has been doing all these years means that she is incompetent and doesn't care about her patients." Which of the following would be the most effective response to Elizabeth? a. "I understand how you feel, but you are going to have to change." b. "It is unfortunate that you feel this way. Others seem quite excited about the new information." c. "It is difficult sometimes to change what we know very well. Sometimes it can be frightening." d. "Perhaps I can arrange some more information sessions for you, so you can see just how important this change is to patient safety."

ANS: C Dialogue can reveal areas where individuals feel inept or overwhelmed, providing the leader with an understanding of what programs need to be developed to increase personal ability to change and what educational initiatives need to be implemented to support change. To promote dialogue, leaders must serve as facilitators, promoting the sharing of ideas, fears, and honest reactions to the change proposal.

After staff meetings lately, Sharon, the head nurse, observes her staff in small groups, having animated discussions that end abruptly when she approaches. Sharon reflects on this observation and realizes that: a. two very outspoken members tend to dominate discussions in meetings. b. this behavior is indicative of a high level of communication among her staff. c. staff members are very committed to the team and have strong opinions. d. ongoing discussion outside of meetings is conducive to creativity.

ANS: C When team communication is dominated by a few members, leaving others uninvolved or bored, disagreement is not expressed openly. As a result, team members "stuff" their feelings and wait until after meetings to voice their opinions.

You anticipate that your region will soon move towards an e-health record system. You begin to discuss this with your staff and are disappointed that you receive little positive response from the staff about this possibility. One staff member, in particular, seems to sum it up by saying "e-health? Won't happen in my working life! There are too many problems with it, like privacy issues." This response is most likely motivated by: a. Lack of urgency regarding the need to change. b. Lack of evidence to support importance of technology. c. Deficits in education and experience. d. Lack of organizational support for change.

ANS: C For leaders to inspire change, they must have intimate knowledge of what matters to the people they manage. Kotter (2012) characterizes this as establishing a sense of urgency, and this involves overcoming complacency. This is especially hard when there doesn't seem to be any visible crisis, or the crisis seems irrelevant to the people being asked to change (Kotter, 2012).

Elizabeth is an example of a(n): a. Early adopter. b. Late majority. c. Laggard. d. Resister.

ANS: C Laggards prefer keeping traditions alive ("We have always done it this way") and openly express their resistance to ideas (speaking out against the change). Late majority individuals are openly negative but will engage with new ideas when most others adopt the change.

17. Justin is a nurse manager in a rehabilitation unit in a small urban center. There is a high turnover rate among rehab-assistants because of the heavy work assignments. Despite his need for staff, Justin decides to review each application thoroughly and interview candidates carefully because he recognizes that it is important to hire staff who can best provide high-quality care and who will fit well with the team. Which of the following decision-making models did Justin use in making his decision? a. Subjective model b. Objective model c. Optimizing model d. Satisficing model

ANS: C Optimizing is a decision style in which the decision maker selects the option that is best, based on an analysis of the pros and cons associated with each option. A better decision is more likely when using this approach, although it takes longer to arrive at a decision.

Team Member A and Team Member B engage in heated disagreements on a frequent basis in team meetings. Their behavior is characterized by insistence on their points of view and refusal to back down or to negotiate alternative solutions once their ideas have been expressed. This behavior is characteristic of: a. autocratic leadership. b. constructive conflict. c. dualism. d. creativity.

ANS: C Our society tends to be dualistic in nature. Dualism means that most situations are viewed as right or wrong, black or white. Answers to questions are often reduced to "yes" or "no." As a result, we sometimes forget a broad spectrum of possibilities actually exists. Exercising creativity and exploring numerous possibilities are important. This allows the team to operate at its optimal level.

The nurse manager used a mediator to help resolve conflicts on the unit. During the mediation process, the nurse manager saw signs of potential team building. One key concept of an effective team is: a. conflict. b. task clarity. c. commitment. d. a designated leader

ANS: C Team building involves moving toward a common vision, which requires commitment. Conflict and clarification of tasks are components in the development of this commitment.

An important aspect of managing the costs on a unit is to plan accurately for staffing needs. Nurse managers use staffing plans to: a. assign staff on the unit on a daily basis. b. ensure that days off are planned for the staff. c. outline the number of individuals by classification on a per-shift basis. d. predict the numbers and classifications of float staff needed to augment regular staff

ANS: C A nurse manager needs to manage financial resources by developing staffing plans. Staffing plans plan for minimum number of professional nurses required on a unit at a given time or to the amount of minimum staffing in an extended-care facility or prison, based on regulatory guidelines.

Scheduling is a function of implementing the staffing plan by assigning unit personnel to work specific hours and specific days of the week. To retain nursing staff, the nurse manager must incorporate into the schedule plan: a. all weekends off. b. all holidays off. c. a variety of scheduling options. d. rotating shifts.

ANS: C Creating a flexible schedule with a variety of scheduling options that leads to work schedule stability for each employee is one mechanism likely to retain staff, which is within the control of nurse managers.

To prepare staffing schedules, a nurse manager needs to calculate paid non-productive time. When calculating paid non-productive time, the nurse manager considers: a. work time, educational time, and holiday time. b. paid hours minus worked hours. c. vacation time, holiday time, and sick time. d. paid hours minus meeting time.

ANS: C Non-productive hours are hours of benefit time and include vacation, holiday, and personal or sick time.

An example of one strategy to improve participation in the change process by staff fitting the behavioral descriptions of laggards, early majority, late majority, and rejecters is to: a. Encourage teamwork. b. Transfer to a different unit. c. Require attendance at staff meetings. d. Delegate the roles and tasks of change.

ANS: D According to Rogers's work, the individual's decision-making actions pass through five sequential stages. The decision to not accept the new idea may occur at any stage. However, peer change agents and formal change managers can facilitate movement through these stages by encouraging the use of the idea and providing information about its benefits and disadvantages.

Elizabeth, an RN with approximately 15 years of service on your unit, walks away from one of the learning sessions on IV care and you overhear her telling a colleague that she thought the session was a waste of time because "the unit has been using heparin for years and there has never been any adverse effects." You follow up with Elizabeth and discover that she is really quite angry about the information sessions because she feels that you are implying that "what she has been doing all these years means that she is incompetent and doesn't care about her patients." Your response to her indicates that: a. Elizabeth will never adopt the change. b. Elizabeth is insecure in her practice. c. Elizabeth requires more information about the practice. d. Change involves emotions.

ANS: D Change, whether proactively initiated at the point of change or imposed from external sources, affects people. Responses to all or part of the change process by individuals and groups may vary from full acceptance and willing participation to outright rejection or even rebellion. It is critical to be able to "read" people and to recognize that communication should involve people's emotions and feelings.

18. Justin is a nurse manager in a rehabilitation unit in a small urban center. There is a high turnover rate among rehab-assistants because of the heavy work assignments. Despite his need for staff, Justin decides to review each application thoroughly and interview candidates carefully because he recognizes that it is important to hire staff who can best provide high-quality care and who will fit well with the team. Which of the following decision-making solutions should Justin consider to have a more efficient department? a. Replace staff only with qualified applicants. b. Determine what the problem or problems are before hiring new staff. c. Consult with the human resources department and develop a plan for hiring new staff. d. All listed are appropriate to consider.

ANS: D Decision making involves a goal-oriented consideration of many options that are objectively weighed according to their possible risks, consequences, and positive outcomes. The options should be ranked in the order in which they are likely to result in the desired goals or objectives. The solution selected should be the one that is most feasible and satisfactory and has the fewest undesirable consequences. In this instance, all of the options listed might be considered and weighed.

12. Sue, a nurse manager, has a staff nurse that has been absent a great deal for the past 3 months. A whistleblower gives some information to Sue indicating that the staff nurse will be resigning and returning to school. Because of this, Sue decides to do which of the following? a. Immediately fire the staff nurse. b. Speak to the whistleblower and elicit more information. c. Speak to the staff nurse and ask her to resign. d. Do nothing.

ANS: D Doing nothing is often warranted because of lack of energy, time, or resources to solve the real problem adequately, and because the benefits are not seen as sufficiently compelling to commit to an action. Do nothing may also be utilized when the individual is not able to do anything for the particular situation or issue.

5. After the nurses who work on an adolescent psychiatric unit have had a brainstorming session, they are ready to resolve the problem of unmanageable teenage patients. To maximize group effectiveness in decision making and problem solving, the nurse manager has: a. prevented conflict. b. formed highly cohesive groups. c. used majority rule to arrive at decisions. d. encouraged equal participation among members.

ANS: D Groups are more likely to be effective if members are involved, the group is cohesive, communication is encouraged, and members demonstrate some understanding of the group process. The nurse leader or manager should provide a nonthreatening and positive environment in which group members are encouraged to participate actively.

1. The risk manager informs the nurse manager of an orthopedic unit that her unit has had an increase in incident reports related to patients falling during the 11-7 shift. The nurse manager knows that the best way to resolve the problem is to: a. use creativity. b. obtain support from the 7-3 shift. c. use institutional research. d. identify the problem.

ANS: D Identification of a problem is the first step in problem solving and occurs before any other step. The most common cause for failure to resolve problems is the improper identification of the problem/issue; therefore, problem recognition and identification are considered the most vital steps.

7. A clinic nurse has observed another nurse deviating from agency policy in performing wound care. The best approach for the clinic nurse to take is to: a. stay out of it. b. inform the nursing supervisor. c. fill out a notification form (incident report). d. assess the risk to the client and the agency before proceeding.

ANS: D If the situation is subjective, nonroutine, and unstructured or if outcomes are unknown or unpredictable, the nurse leader and manager may need to take a descriptive or behavioral approach. More information (such as degree of risk to the client and to the agency) needs to be gathered to address this situation effectively.

As the unit manager, you spend a day performing direct patient care and work with a new system that is designed to capture patient documentation at the bedside. During discussions with staff while giving care, you discover that the number of screens that need to be opened during documentation makes charting more complex and time-consuming than traditional manual charting approaches. On the basis of this feedback, you: a. Assume that the system is doing what it needs to do. b. Provide reassurance to staff that the unit has achieved its goals in implementation of the system. c. Ask some of the staff if they have had similar experiences with the system. d. Consult chart audit data and end user consultation reports to determine if errors and problems are occurring.

ANS: D In Kotter's eight-step change model, removing obstacles means keeping alert for barriers in structure and processes that limit the ability to change and then removing those barriers once they have been found.

The wound care nurse decided to involve those to be affected by change early in the change management process. This can positively result in: a. Coordination. b. Resistance. c. Anticipation. d. Participation.

ANS: D Successful change means persistence and advancement of the change, which requires the undivided focus of all team members. Early involvement and participation are critical to capturing the undivided focus of team members.

Elizabeth, an RN with approximately 15 years of service on your unit, walks away from one of the learning sessions on IV care and you overhear her telling a colleague that she thought the session was a waste of time because "the unit has been using heparin for years and there has never been any adverse effects." According to Havelock (1973), this comment may originate from failure in which phase of the six phases of planned change? a. Generating self-renewal b. Choosing the solution c. Diagnosing the problem d. Building a relationship

ANS: D The first phase of this model of planned change involves building a relationship as a basis for later phases, which include diagnosing the problem and choosing the solution. At this point, as a new manager, the relationship may not have yet developed sufficiently with Elizabeth.

24. The agency in which you are a nursing leader makes a decision to reduce the number of RN positions in favor of PN positions because agency data suggest that the clients in the agency can receive appropriate care from PN staff. Furthermore, the agency is facing a decline in funding and without restructuring, some clients might not receive services at all. You provide this information on this decision to the staff and ask them to advise you if they have any feedback, concerns, or alternative solutions. This decision-making style is known as: a. paternalistic. b. shared. c. reasoned d. informative.

ANS: D The informative model offers the staff the ability to make a decision after the information has been shared and without the active involvement of the manager.

19. When confronted with the controversy and the apparent poor morale of the evening staff, the unit manager decided the staff needed to take some time off. He scheduled holidays for the staff without consulting them. A couple of the staff nurses approached the manager and indicated that the problem was not scheduling, but rather the team leader and her patient assignments. What was the unit manager's first missed step in problem solving? a. Not using a problem-solving model b. Not considering a number of alternatives c. Poor evaluation of outcomes d. Incorrect problem identification

ANS: D The unit manager did not begin with an accurate identification of the problem. Problem solving needs to begin with "why?".

23. Jane, an experienced head nurse, is given the task of completing the summer vacation schedule for the pediatric unit. She is fully aware of the hospital's restrictions on time off and the number of staff on vacation at any given time, as well as its issues regarding seniority. She weighs the options of allowing staff choice, such as it takes more time but gives employees options. However, if choice is allowed, this could cause arguments. Which of the following is the best alternative? a. Ask for requests for vacation time in advance, and post the times. b. Post the completed vacation schedule. c. Post a tentative schedule, and request feedback. d. Post a blank schedule, and ask staff members to fill in their times by a given date.

ANS: D This is based on a decision-making model that allows experience and knowledge to predict whether a decision will or will not work. The experience of the head nurse suggests that it is important to involve staff in decisions that affect them the most.

8. In a rural hospital, the unit for which you are charge nurse has a particularly busy morning. A 52-year-old patient is complaining of left-sided chest pain and a multiparous patient is about to deliver. A child with asthma is experiencing early signs of an attack. The other RN on the unit is a recent graduate who has not yet been orientated to the labor room and has limited cardiac nursing experience. An unlicensed assistant is available to assist. You must decide which patient situation you will take and where the new graduate RN's skills can best be used. Given the limitations in skills and experience, number of staff available, and time constraints, you must make a decision that involves: a. a higher order thinking process. b. selecting the best option for reaching a predefined goal. c. optimizing. d. satisficing.

ANS: D With this approach, the decision maker selects an acceptable solution, one that may minimally meet the objective or standard for a decision. This approach allows for quick decisions and may be the most appropriate when lack of time is an issue.

A nurse manager is experiencing conflicts between herself and staff members. She had tried to develop a team by using a shared leadership model to empower the staff. Staff members are functioning: a. as a team. b. independently. c. interdependently. d. as a group.

ANS: D A group is a collection of interconnected individuals working together, with a high degree of interdependence, for the same purpose. A team is a unified group that is committed to a common purpose, performance goals, and approach, for which they hold themselves mutually accountable. The conflict indicates that the staff may not be united in a common purpose.

The staff development educator developed strategies to help nurse managers actively listen. Guidelines for active listening include which of the following? a. Speed up your internal processes so that you can process more data. b. Realize that the first words of the sender are the most important. c. Be prepared to make an effective judgment of the communication sender. d. Cultivate a desire to learn about the other person.

ANS: D Active listening means suspending judgment about what is about to be said and listening to all that is said (and not just the first or last words). It is motivated by a genuine desire to learn about the other person

The nurse manager was upset with the staff nurse and said, "You did not understand what I said." Which element in the communication process was she referring to? a. Feedback between receiver and sender b. A message channel c. A receiver who decodes the message d. A set of barriers that may occur between sender and receiver

ANS: D Problems can occur at any point in communication and result in miscommunication. In this instance, it can be assumed that there was a sender, a receiver, a channel, and feedback. In this scenario, barriers such as distractions, inadequate knowledge, differences in perceptions, and emotions and personality may have resulted in misunderstanding between the manager and the staff nurse.

A nurse manager must consider a number of external variables when preparing the personnel budget and projecting the unit's staffing needs. An external variable to be considered is: a. organizational staffing policies. b. staffing models. c. changes in services that will be offered. d. department of Health licensing standards.

ANS: D Licensing regulations of the state can determine staffing models. Staffing regulations can dictate the number of professional nurses required on a unit at any given time.

Second-order change

Change that is revolutionary, episodic, and that requires radical differences form what exists. May be unanticipated or expected.

Planned change

Deliberate and organized and has the goal of improvement. Change expected and deliberatively prepared beforehand by using systematic directional processes to develop and carry out activities to accomplish a desired outcome.

Change leaders

Those who create the vision and foster major organizational transformation.


Conjuntos de estudio relacionados

section 12 unit 1: The Loan Process and Qualifying the Borrower

View Set

Med-Surg Final Exam Review Questions

View Set

Chapter 37: Management of Patients With HIV Infection and AIDS

View Set

fahmy 1 ( bible ) English& Greek 10

View Set

Psychology Module 46- The Biomedical Therapies and Preventing Psychological Disorders

View Set

HUN-2202 Chapter 5 Quiz 5A (Practice)

View Set

Alabama Real Estate Final Exam Study Guide - 60 Hour Course

View Set

N1 Goi Tettei Toreningu Full (2)

View Set

real estate principles: chapter 6

View Set